MENTAL HEALTH FINAL REVIEW

Réussis tes devoirs et examens dès maintenant avec Quizwiz!

CH 32. Which statement by a young client who has severe and persistent mental illness would alert the nurse to the need for psychoeducational intervention? "I like to watch cartoons every morning." "I hear that marijuana helps calm you down." "I am looking for a job washing dishes at a diner." "I hate having my thoughts so messed up all the time."

"I hear that marijuana helps calm you down." - Clients with mental illness should receive information about the dangerous negative impact of using illegal drugs. None of the other options suggest situations that are unsafe for the client.

Overt statement

"I wish I was dead" "I cant take it anymore" "Everyone would be better off if i died"

Covert statements

"It's okay, now. Soon everything will be fine." "Things will never work out." "I won't be a problem much longer." "Nothing feels good to me anymore and probably never will." "How can I give my body to medical science?"

CH 29. A sexual assault survivor tells the nurse, "I should have tried to fight him off! But I was so terrified that I could not move. I should have tried harder." Which response should the nurse make to reassure the client? "Try not to think about it. Put it out of your mind." "We each behave in characteristic ways in a crisis. That was your way." "Do you think others will think badly of you for not trying to fight?" "The way you behaved was the right thing to do at the time."

"The way you behaved was the right thing to do at the time." - The victim should always be told that staying alive was the priority and that whatever she did to that end was the right thing to do. None of the other options provide reassurance as effectively as the correct option

CH 29. During the immediate post-rape period what verbal nursing intervention would best lower client anxiety and increase feelings of well-being? "You are safe here. I will stay with you while you have your examination." "I know you feel confused. We will make all the necessary decisions for you." "Please tell me as much about the details of the rape as you can remember." "When you leave you will be given follow-up appointments for pregnancy and sexually transmitted disease screening."

"You are safe here. I will stay with you while you have your examination." - The presence of the nurse is reassuring, especially when the client is experiencing disorganization and the environment is confusing. Safety is the primary concern for both the client and the nurse. The nurse's presence conveys a sense of safety to the client. None of the other options address safety and well-being.

Depersonalization

(focus is on oneself, being an observer in one's own body or mental processes) derealization (focus is on the outside word, one's surroundings are unreal or distant) disorder Feels distressed,.....mechanical, dreamy or detached from the body

Maslow's Hierarchy of Needs

(level 1) Physiological Needs (level 2) Safety and Security (level 3) Relationships, Love and Affection (level 4) Self Esteem (level 5) Self Actualization

what is the most important assessment for someone with major depression?

**Assessment of Suicide Potential** - most important, practice asking in nonchalant but genuine way, ask as if you are asking simply for the s/s they are experiencing

Difference between Dysthymia and major depression?

**However, does NOT interfere with functioning - CAN still hold job, have friends, be social, etc. **NO suicidal ideation**

do to the long period of time it takes for TCAs to take effect, what is recommended for these pts?

**Partial programming** - is best way to monitor patient after hospital stay and crisis intervention

Hospital Setting Interventions for anger and aggression Clients with healthy coping

*Clients with healthy coping who are overwhelmed* - Acknowledge client distress - Validate feeling as understandable - Search for solutions - Name the feeling the client can't articulate :Discounted, embarrassed, frightened, guilty, humiliated, hurt, ignored. Not heard, rejected...

Nursing considerations for DELIRIUM?

*Need to catch underlying cause to prevent irreversible brain damage* Acute delirium almost always requires one-to-one Do not challenge their delusions or hallucinations but try to re-orient them Give optimistic but realistic reassurance Ask Concrete Questions - nothing that requires abstract thinking Can encourage visitors if it will calm Remove negative stimuli

Patient Self-Determination Act

- 1990 - can indicate directions for type and amount of care they would receive if/when they cannot make their own decisions - need to have copy in patient chart

what are some examples of where individuals seek help when experiencing a crisis?

- 23 hour observation - short term crisis residential services - 24/7 crisis hotline - warm lines - mobile crisis services - peer crisis services - psychiatric advance directive plans

Medications used for Conduct Disorder?

- 2nd gen antipsychotics to control behaviors, Geodon/Zyprexa - tricyclics, anxiolytics, mood stabilizers to manage aggression

PTSD Epidemiology and comorbidities

- 55-90% of people have experienced a traumatic event! - Following a traumatic event 8% will develop PTSD - Lifetime prevalence of PTSD is 35% - Comorbidities include: major depressive disorder, anxiety disorders, sleep disorders, and dissociative disorders.

how often will pt receive ECT?

- 8 to 12 treatments scheduled 3x / week

Outcome Identification for sexual assault?

- Absence of any residual symptoms after the trauma - Abuse protection - Abuse recovery: emotional - Abuse recovery: sexual - Coping - Personal resiliency - Sexual functioning - Stress level

More Common Extrapyramidal Symptoms (EPS)

- Acute dystonia - Acute, sustained contraction of muscles, usually of head and neck - let doctor know. This is bad. - Akathesia - restless. Cant stop moving - Pseudoparkinsonism - can have tremors, trouble starting or stopping walking. Pill rolling

outcome identification for Phase II Stabilization in schizophrenia

- Adhere to treatment - Stabilize medications - Control or cope with symptoms

Characteristics of Intermittent explosive disorder

- Adults 18 years or older - Leads to problems with - Interpersonal relationships - Occupational difficulties - Criminal difficulties Tension and arousal Aggression & explosive behavior Remorse, regret, embarrassment

Eugen Bleuler's - 4 A's of Schizophrenia

- Affect - they commonly have a flat affect - Associative looseness - Autism - out of touch with reality. - Ambivalence - two opposing emotions or attitudes. Positive and negative thoughts about something.

What are the symptoms of major depression?

- Anhedonia - fatigue - sleep disturbances - changes in appetite - feelings of hopelessness or worthlessness - persistent thoughts of death or suicide - inability to concentrate or make decisions - change in physical activity - Constant inability to have a stable mood

treatment for EPS

- Anticholinergic drug: benzatropine (Cogentin), artane, Benadryl - Switch to atypical antipsychotic?

Adverse effect of TCAs?

- Anticholinergic effects - sedation and postural hypotension can lead to falls - urinary retention and constipation warrant IMMEDIATE MEDICAL ATTENTION

Potentially Dangerous &/or Life-threatening Responses to Antipsychotics

- Anticholinergic toxicity - Neuroleptic malignant syndrome (NMS) Can occur with 1st gen. meds, occasionally with 2nd gen meds - Agranulocytosis

Possible Nursing Diagnoses for Anxiety

- Anxiety (level) - Ineffective coping - Disturbed thought processes - Post trauma syndrome - Insomnia - Hopelessness - Self care deficit - Impaired skin integrity - Imbalanced nutrition

what is sexual assault?

- Any type of sexual activity the victim does not want or agree to - From inappropriate touching to penetration - Verbal sexual assault can occur by phone or electronically online - Forced activities - Prostitution or posing for pornographic pictures or videos - Children, older adults, women, or men can be victims

give examples of 3rd gen antipsychotics

- Apriprazole (Abilify) - Brexpiprazole (Rexulti) - Cariprazine (Vraylar)

3rd generation antipsychotics

- Aripiprazole (Abilify) - Dopamine system stabilizer - Improves positive and negative symptoms and cognitive function - Little risk of EPS or tardive dyskinesia - Better safety profile than 2nd gen, but may be less effective than some.

Implementation/Intervention for Mild to Moderate level of anxiety

- Ask open ended questions - Provide calm presence - Evaluate coping mechanisms - Help patient consider alternatives to problem situation - Offer activities & methods to relieve tension

Documentation of a Violent Episode includes?

- Assess preassaultive behaviors - Nursing interventions & client response - Describe client behaviors during assault - Interventions used to diffuse crisis and client response - Client behavior and staff interventions while client in restraints - The way client reintegrated into milieu

Rationalization (defense mechanism)

- Attempting to make excuses or formulate logical reasons to justifying unacceptable feelings or behaviors. EX: John tells the rehab nurse, "I drink because its the only way I can deal with my bad marriage and awful job."

projection (defense mechanism)

- Attributing feelings or impulses unacceptable to ones self to another person. EX: Sue feels a strong sexual attraction to her track coach and tells her friend, "Hes coming on to me!"

2nd generation antipsychotics

- Atypical antipsychotics - less side effects - Seratonin-dopamine antagonists - Treat both positive and negative symptoms - Minimal to no extrapyramidal side effects (EPS) or tardive dyskinesia

dissociative disorders Epidemiology and Etiology

- Believe to be rare - Dissociative Disorder: Stats: 2-10% Etiology: - Childhood, physical, sexual or emotional abuse and other traumatic life events - Acute overwhelming trauma

Implementation/Interventions for Severe to Panic level of anxiety

- Calm demeanor; Firm, short statements - Provide safety for patient and others - Ensure personal needs met (food, rest etc.) - Decrease stimulation - Medication (scheduled or prn) - Restraints (prn) - least restrictive first - Reality orientation

Assaultive Stage: Seclusion and Restraint

- Can only be used if client is a danger to self or others -Can only be used when less-restrictive methods have been tried and failed -Require a physician's order and assessment - Order includes reason client has been confined, how he or she should be confined, maximum time of confinement, and criteria for release - Client must be directly observed by a staff member for safety during the entire confinement - Licensing and accreditation agencies guide the times for assessment, hydration, toileting needs, nutrition, comfort, and documentation - They need to be placed in a locked room so no one else can come in and hurt them while they're restrained

alterations in behavior in schizophrenia (positive)

- Catatonia - increase or decrease in the rate and amount of movement - Motor retardation - slowed - Motor agitation - excited behavior such as running or pacing rapidly - Stereotyped behaviors - repetitive behaviors that do not serve a logical purpose - Echopraxia - the mimicking of movements of another - Boundary impairment - not good boundaries, might say inappropriate things - Automatic obedience - do whatever you tell them to do - Waxy flexibility - stay in a position until you move it. usually seen in catatonia - Negativism - tendency to oppose or resist the requests or wishes of others - Impaired impulse control - Gesturing or posturing - assuming unusual and illogical expressions (often grimacing) or positions

Substance-induced anxiety disorder

- Characterized by symptoms of anxiety, panic attacks obsessions, and compulsions that develop with use of a substance - Develop with the use of a substance or within a month of stopping use of the substance

give examples of 2nd gen antipsychotics

- Clozapine (Clozaril) - risperidone (Risperdal) - olanzapine (Zyprexa) - Quetiapine (Seroquel) - Ziprasidone (Geodon)

what are some characteristics of perpetrators?

- Consider their own needs more important than needs of others - Poor social skills - Extreme pathological jealousy - May control family finances - Likely to abuse alcohol or drugs

outcomes for pt. in crisis

- Coping - Decision making - Role performance - Stress level

Compensation : Defense Mechanism

- Covering up a real or perceived weakness by emphasizing a trait one considers more desirable. EX: A handicapped boy is unable to participate in football, so he compensates by becoming a great scholar.

what are the symptoms of adjustment disorder?

- Debilitating cognitive, emotional, and behavioral symptoms that negatively impact normal functioning - Responses to stressful event may include combinations of depression, anxiety, and conduct disturbances - Symptoms: - guilt, depression, anxiety, physical complaints, social withdrawal

SIDE EFFECTS of ADHD medications?

- Decreased Appetite - headaches - abdominal pain - Insomnia - jitteriness - Social withdrawal

depersonalization/derealization dissociative disorder

- Depersonalization (focus is on oneself, being an observer in one's own body or mental processes) - derealization (focus is on the outside word, one's surroundings are unreal or distant) disorder Feels distressed,.....mechanical, dreamy or detached from the body

alterations in perception in schizophrenia (positive)

- Depersonalization - feeling of being unreal or having lost identity - Derealization - environment is not real or has changed - Hallucinations - Auditory - most common - Command - Visual - second most common - Tactile - feel things touching them - Gustatory - tasting things that are not there

Common comorbidity with ADHD?

- Depression

cognitive symptoms of schizophrenia (negative)

- Difficulty with (lack of) - Attention - Memory - Cognitive flexibility - Executive functions - day to day things - Thought blocking - Concrete thinking (lack of ability to think abstractly)

nursing diagnoses for positive symptoms in schizophrenia

- Disturbed sensory perception - Risk for self-directed or other-directed violence - Disturbed thought processes

disadvantages of 1st gen. antipsychotics

- Do not treat negative symptoms - Extrapyramidal side effects (EPSs), Tardive dyskinesia - Anticholinergic side effects - Lower seizure threshold - Meds can throw them into a seizure - Weight gain, sexual dysfunction, endocrine disturbances

Examples of TCAs?

- Elavil - Norpramin - Sinequan - Tofranil - Pamelor

assessment interventions for schizophrenia

- Empathy - Minimize questioning - Direct but supportive - Indirect therapeutic confrontation - Obtain, validate data from others - Prioritize data sought

Tertiary prevention for abuse

- Facilitating healing and rehabilitative process - Providing support - Assisting survivors of violence to achieve their optimal level of safety, health, and well-being

Eastern Tradition

- Family basis for identity - Body-mind-spirit one entity - Disease caused by fluctuations in opposing forces - Time is circular and recurring - Born into a fate; duty to comply

Health teaching for substance abuse? - F.R.A.M.E.S.

- Feedback of personal risk - Responsibility of the patient - Advice to change - Menu of ways to reduce substance use - Empathetic counseling - Self-efficacy or optimism of the patient

Common date rape drug

- Flunitrazepam (Rohypnol or "roofies") - Gamma hydroxybutyric acid (GHB)

Psychopharmacology: Alzheimers ~ Tacrine/Cognex

- For mild and moderate symptoms, improves functioning and slows the progress of the disease in the areas of cognition and memory. - High frequency of side effects including elevated liver transaminase levels, GI effects and liver toxicity. START LOW GO SLOW

Give examples of 1st Gen antipsychotics

- Haloperidol (Haldol) - chlorpromazine (Thorazine) - Decanoate (Injectable)

Assessing coping styles for stress (nursing management)

- Health sustaining habits (e.g., medical compliance, proper diet, relaxation) - Life satisfactions (e.g., work, family, hobbies, humor, art, nature) - Social supports - Effective and healthy responses to stress

Dissociative Disorders Assessment

- History - Moods - feeling unreal, perceiving world as unreal, losing track of time - Impact on patient and family - Suicide risk - Self-assessment

Potential Nursing Diagnoses for Crisis Intervention

- Ineffective coping - Anxiety - Risk for suicide - Social isolation

Introjection: Defense Mechanism

- Integrating the beliefs and values of another individual into ones own ego structure. EX: Children integrate their parents value system into the process of conscience formation.

Dissociative Disorders Implementation

- Integration takes time - Being present during the recall of painful experiences - Providing sense of safety - Encouraging optimal level of functioning - Psychoeducation (coping skills and stress management, grounding techniques) - Pharmacological interventions - Advanced practice interventions - Somatic therapy (body, mind, emotions and spirit are interrelated)

Repression (defense mechanism)

- Involuntary blocking of unpleasant feelings and experiences from ones awareness. EX: An accident victim can remember nothing about the accident

evaluation in schizophrenia treatment

- Is patient responding to meds? - Able to concentrate more? - Still having hallucinations/delusions? - Are med s/e problematic? - How are they handling meds and their side effects? - If little or no progress? - reassess - Cooperation with treatment?, need for different intervention? - (per Recovery Model) ask patient for input about why outcome has not occurred. - Cheeking meds

what should an abuse assessment include?

- It should include violence indicators, level of anxiety, support systems, do they have suicidal/homicidal tendencies, is there substance abuse, self-assessment.

pts with Antisocial PD are most commonly in?

- JAIL

Hormone imbalances with major depression?

- LOW serotonin - LOW norepinephrine

what do you do if verbally abusive pts. continue to be verbally abusive?

- Leave room when abuse begins "you can say whatever you would like but I will not sit here and listen" - Take neutral stance in interactions "I can understand how you feel" - Withdraw attention when pt abusive, give positive attention when pt nonabusive

Areas of brain involved with aggressiveness?

- Limbic system - responsible for combining higher mental functions and primitive emotions into one system, learning, and the formation of memories - Amygdala - emotional center of the brain, helps evaluate emotional context of our experiences. fight-or-flight - Temporal lobe

young adults with serious mental illness

- Limited experience with formal treatment - Contribute to a cycle of treatment, brief recovery, and relapse - Additional problems, such as increased frequency of relapse, legal difficulties, homelessness, substance abuse, and unemployment

alterations in speech in schizophrenia (positive)

- Loose associations - no connections - Neologisms - newly coined word or expression - Echolalia - repeating what youre saying - Clang associations - saying things and rhyming - Word salad - bunch of loose associates. Mumbo jumbo

Examples of Hallucinogens?

- Lysergic acid diethylamide (LSD or acid) - Mescaline (peyote) - Psilocybin (magic mushroom) - Phencyclidine piperidine (PCP, angel dust, horse tranquilizer, peace pill)

outcome identification for Phase III Maintenance in schizophrenia and planning

- Maintain achievement - Prevent relapse - Achieve independence, satisfactory quality of life planning: Provide patient and family education Relapse prevention skills are vital

Outcome identification for PTSD

- Manage anxiety as demonstrated by use of relaxation techniques, adequate sleep, and ability to maintain role or work requirements - Increase self-esteem - Improve ability to cope

Predisposing factors for Impulse control disorders?

- May be a genetic component - Less gray matter in emotional regions of brain - Different levels of serotonin & testosterone - ADHD - Behaviors may compensate for low self esteem

why is assessment for depression crucial in schizophrenia?

- May herald impending relapse - Increases substance abuse - Increases suicide risk - Further impairs functioning

assessment guidelines for schizophrenia

- Medications & medical concerns - Presence and severity of positive and negative symptoms - Hallucinations - Delusions - Family & patient's knowledge of patient's illness and symptoms - Comorbidities - Belief system/delusions - Co-occurring disorders - Abuse of or dependence on alcohol or drugs - Risk to self or others - Command hallucinations

Interventions for Stabilization Phase II for schizophrenia

- Monitor and document Hallucinations, Delusions - Milieu management and Safety - Activities and groups - Health teaching and health promotion - Counseling and communication techniques

Rape-Trauma Syndrome: Acute Phase

- Occurs immediately after the assault - May last for a few weeks - Lifestyle disorganized - Somatic symptoms are common - Reaction to crisis includes disruptions in cognitive, affective, and behavioral functions - Rape is an adventitious crisis

Characteristics of Borderline PD?

- Overwhelming needs -which they seek to have met in relationships - Their glass will NEVER be full - never enough love/attention/etc. - Major defense is SPLITTING - people are either all good or all bad - Intense relationships - Impulsivity, self-mutilation - Chronic emptiness, pervasive anger - Intense fear of abandonment - Borderline PD behavior

Outpatient Psychiatric Mental Health Care

- PRIMARY CARE PROVIDERS Specialty psychiatric care providers Patient-centered health/medical homes Community clinics Psychiatric home care Assertive community treatment (ACT) Partial hospitalization programs (PHPs)

When do you consider that treatment is effective for PTSD?

- Patient recognizes treatment is effective - Patient is able to use newly learned strategies to manage anxiety - Pt. experiences no flashbacks - Able to sleep without nightmares - Assume work/family roles

Assessment of pt with ANOREXIA includes?

- Patient's perception of the problem - Eating habits - very detailed about what their meals consist of - History of dieting - there is usually a history of trying many different diets/supplements - Methods used to achieve control - exercise/laxatives/diuretics - Value attached to a specific shape and weight - ask them what their "danger weight" is; ask them what their ideal weight is - Interpersonal/social functioning - friendships/parents/siblings - Mental status - Medical instability - vitals/EKG

incidence of rape?

- People of lower socioeconomic classes more often victims - 80% of victims are white - 1/3 of all assaults occur inside home - Most of time, no weapon involved - Young females (12-19) at particular risk - 9% to 10% of all rapes are male victims - 1 in 5 women and 1 in 71 men in the United States have been raped at some time in their lives

Alterations in Thinking in Schizophrenia (positive)

- Persecution - believing one is being singled out for harm - Grandiosity - believing one is a very powerful or important person - Religiosity - Somatic sensations - believing body is changing in unusual ways - Delusion of being controlled - Ideas of reference - Thought broadcasting - everyone knows that you are thinking - Thought insertion - someone else has inserted their thoughts into your brain - Thought withdrawal - Hypochondriasis

Characteristics of Narcissistic PD?

- Pervasive pattern of grandiosity - Fantasies of brilliance or power - Need to be admired - Sense of entitlement - Arrogant, patronizing, and rude

Characteristics of Vulnerable Persons: Older Adults

- Poor mental or physical health - Dependent on perpetrator - Female, older than 75 years, white, living with a relative - Elderly father cared for by a daughter he abused as a child - Elderly woman cared for by a husband who has abused her in the past

Characteristics of Vulnerable Persons: Women

- Pregnancy may trigger or increase violence - Violence may escalate when wife makes move toward independence - Greatest risk for violence when attempts to leave relationship

Psychopharmacology: Alzheimers ~ Namenda

- Prescribed in the moderate to severe stages of the disorder. - Namenda is a unique Alzheimer's disease medication that has a low rate of side effects and has been proven effective in helping to treat the symptoms of Alzheimer's disease when taken alone or as part of a combination therapy.

reaction formation (defense mechanism)

- Preventing unacceptable thoughts or behaviors from being expressed by exaggerating opposite thoughts or types of behaviors. EX: Jane hates nursing. She attended nursing school to please her parents. During career day, she speaks to prospective students about the excellence of nursing as a career.

assessment for PTSD

- Primary Care PTSD Screen (PC-PTSD) - PTSD Checklist (PCL-5) - Need a comprehensive assessment! - History of time of onset, frequency, course, severity, level of distress, and degree of functional impairment - Suicidal? Homicidal? - Social supports? - Insomnia? - Social Withdrawal? - Current life stressors, medications, past medical and psychiatric history, and mental status exam

define "crisis"

- Profound disruption of normal psychological homeostasis - Normal coping mechanisms fail - Results in inability to function as usual - Threatens personality organization - Presents opportunity for personal growth - Acute and time-limited

neurotrophic factors

- Proteins - Gases Proteins responsible for growth and survival of neurons during development and maintaining adult neurons

follow up after initial eval. for sexual assault?

- Provide follow-up assessment within 24 to 48 hours - Provide follow-up visits at 2, 4, and 6 weeks

handling clients with cognitive deficits with anger and aggression?

- Provide means to assist client with reality orientation - Medication - Use short simple sentences - Present calm, pleasant demeanor - Decrease sensory stimulation

Interventions for Acute Phase I for schizophrenia

- Psychiatric, medical, and neurological evaluation - Psychopharmacological treatment - Support, psychoeducation, and guidance - Supervision and limit setting in the milieu

interventions for PTSD treatment

- Psychoeducation - Psychopharmacology (SSRI) - Trauma focused psychotherapy - Imagery - Relaxation techniques - Hypnosis - Group Therapy

Sexual Assault Nurse Examiners (SANEs)

- RNs with specialized training in caring for sexual assault patients - Demonstrated competency in conducting medical and legal evaluations - Ability to be an expert witness in court

outcomes of crisis depends on?

- Realistic perception of the event - Adequate situational supports - Crisis intervention - Adequate coping mechanisms

Informed-Approach intervention

- Realizes the widespread impact of trauma and understands potential paths for recovery; - Recognizes the signs and symptoms of trauma in clients, families, staff, and others involved with the system; - Responds by fully integrating knowledge about trauma into policies, procedures, and practices; and - Seeks to actively resist re-traumatization."

sublimation (defense mechanism)

- Rechanneling of drives or impulses that are personally or socially unacceptable into activities that are constructive. EX: Mom of son killed by drunk driver, president of MADD.

Denial (defense mechanism)

- Refusing to acknowledge the existence of a real situation or the feelings associated with it. EX: A women drinks alcohol every day and cannot stop, failing to acknowledge that she has a problem.

Regression (defense mechanism)

- Responding to stress by retreating to an earlier level of development and the comfort measures associated with that level of functioning. EX: When 2 year old Jay is hospitalized for tonsillitis he will drink only from a bottle, although his mother states he has been drinking from a cup for over 6 months.

patients rights under the law

- Right to treatment - Right to refuse treatment - Right to informed consent - Rights surrounding involuntary commitment and psychiatric advance directives - Rights regarding restraint and seclusion - Right to confidentiality - Right to privacy

Antidepressant Pharmacological interventions for anxiety

- SSRIs are the first line of defense in most anxiety/obsessive disorders. (Paxil, Prozac, Lexapro, Luvox, Zoloft). - SNRI (Effexor, Cymbalta(generalized anxiety disorder)) - MAOIs Monoamine oxidase inhibitors - reserved for Tx-resistant conditions bc of risk of life-threatening hypertensive crisis if pt. does not follow dietary restrictions

Long-term management of chronic aggression

- SSRIs, lithium, anticonvulsants, benzo's, second-gen antipsychotics, and beta blockers

planning for anxiety disorder treatment

- Select interventions that can be applied in appropriate setting for person with anxiety or anxiety disorder. - Relate interventions to level of anxiety capabilities - If possible include patient in planning

Neurotransmitter Stress Responses

- Serotonin synthesis more active - May impair serotonin receptor sites and brain's ability to use serotonin

Serotonin on stress response

- Serotonin synthesis more active during times of stress - May impair serotonin receptor sights and brain's ability to use serotonin

social anxiety disorder

- Severe anxiety or fear provoked by exposure to a social or a performance situation that will be evaluated negatively by others

nursing diagnoses for negative symptoms in schizophrenia

- Social isolation - Chronic low self-esteem - Ineffective coping

Inpatient admission is reserved for individuals who are...

- Suicidal - Homicidal - Extremely disabled and in need of short-term acute care

Undoing: Defense Mechanism

- Symbolically negating or canceling out an experience that one finds intolerable. EX: Joe is nervous about his new job and yells at his wife. On his way home he stops and buys her flowers.

Anxiety due to a medical condition

- Symptoms of anxiety are direct physiological result of a medical condition such as hyperthyroidism, pulmonary embolism, or cardiac dysrhythmias.

psychosocial interventions for anger and aggression

- Tell patient you are concerned and want to listen - State expectations for patient's behavior "its okay for you to be angry, but you cant throw things" - Respect patient's personal space - Speak slowly and in short sentences - Lower your voice - Pay attention to environment; choose quiet place visible to staff for safety - Do not want to go into a closed room with a patient by yourself, and if you do, let someone know. You should always be closest to the door.

Disadvantage of 2nd gen antipsychotics

- Tendency to cause Metabolic Syndrome - a lot of people were becoming diabetic - significant weight gain, altered glucose metabolism - sedation, sexual dysfunction, seizures, increased mortality in elderly with dementia

Trauma-Specific Interventions

- The survivor's need to be respected, informed, connected, and hopeful regarding their own recovery - The interrelation between trauma and symptoms of trauma such as substance abuse, eating disorders, depression, and anxiety - The need to work in a collaborative way with survivors, family and friends of the survivor, and other human services agencies in a manner that will empower survivors and consumers

displacement (defense mechanism)

- The transfer of feelings from one target to another that is considered less threatening or that is neutral. EX: A client is angry at his doctor, does not express it, but becomes verbally abusive with the nurse.

Suppression: Defense Mechanism

- The voluntary blocking of unpleasant feelings and experiences from ones awareness. EX: "I don't want to think about that now. I'll think about that tomorrow."

What is an advantage of TCA medications?

- They are great for pts who are already tired and lethargic from their depression ; they do not cause drowsiness

what defines a serious mental illness? (SMI)

- Those who are most deeply affected by psychiatric disorders - Affects 5% to 7% of U.S. population - Presents difficulties in multiple areas - Associated issues of poverty, stigma, unemployment, and inadequate housing - Chronic or recurrent - Patients at risk for multiple physical, emotional, and social problems - Varying degrees of disability

Rape

- Type of sexual assault - Any penetration of the vagina or anus with any object or body part, or the oral penetration by a sex organ of another person - Or by threat of bodily harm or when a person is incapable of giving consent - Second most violent crime (FBI, 2008)

1st generation antipsychotics

- Typical, Conventional antipsychotics or neuroleptics - bad side effects - Dopamine antagonists (D2 receptor antagonists) - Haldol, prolixin (fluphenazine), navane (thiothixine), thorazine (chlorpromazine), mellaril (thioridazine) - test will show both. Need to know category - Target positive symptoms of schizophrenia - first ones that came out. Helped with hallucinations and delusions, but wasn't helpful for other symptoms

what are some examples of SNRIs

- Venlafaxine (Effexor) - Duloxetine (Cymbalta)

Assessment: Interview Guidelines for abuse - Open ended questions for parents

- What arrangements do you make when you have to leave your child alone? - How do you discipline your child? When your infant cries for a long time, how do you get him/her to stop? - What about your child's behavior bothers you the most? - Don't want to ask questions that sound like youre blaming them or accusing them, want to remain neutral.

Characteristics of Vulnerable Persons: Children

- Younger than 3 years - Perceived as different - Remind parents of someone they do not like - Product of an unwanted pregnancy - Interference with emotional bonding between parent and child

Short-Acting Sedative-Hypnotic Sleep Agents ("Z-hypnotics") for anxiety

- Zolpidem (Ambien)

define "anxiety"

- a universal human experience among the most basic of emotions. apprehension, uneasiness, uncertainty, or dread from real or perceived threat. It is a vague sense of dread related to an unspecified or unknown danger.

how soon does acute stress disorder resolve?

- after a period of a month

FREUD psychosocial factor

- aggression turned inward

Stage 2. Resistance

- also called the adaptive stage - during this time, sustained and optimal resistance to stressor occurs - recovery, repair, and renewal may occur - at this point, individuals have used up valuable resources and have reduced defenses and adaptive energy

what are the symptoms of acute stress disorder?

- alterations in concentration - anger - dissociative amnesia - headache - irritability - nightmares

6 negative symptoms of schizophrenia

- anhedonia - lack of pleasure in life - avoliition - lack of motivation, lack of goal-directed behavior - asociality - lack of social interaction - affective blunting - lack of affect apathy - lack of interest in activities or beliefs that would otherwise be interesting or important - alogia - reduction in speech

advanced practice interventions for anxiety: COGNITIVE THERAPY AND COGNITIVE RESTRUCTURING

- based on belief that pts. Make errors in thinking that lead to mistaken negative beliefs about self and others. - Cognitive restructuring - therapist helps pt. 1. Identify automatic negative beliefs that cause anxiety 2. Explore the basis for these thoughts 3. Reevaluate the situation realistically 4. Replace negative self-talk with supportive ideas

Harry Stack Sullivan anxiety disorder theory

- believed anxiety is linked to emotional distress caused when early needs go unmet or disapproval is experienced (interpersonal theory). Also suggested anxiety is contagious, transferring from mother to infant.

how do 3rd gen antipsychotics work?

- by reducing dopamine in some brain regions while increasing it in others

what are anxiety/OCD medications for children?

- clomipramine (Anafranil) - fluoxetine (Prozac) - fluvoxamine (Luvox) - sertraline (Zoloft)

advanced practice interventions for anxiety: COGNITIVE-BEHAVIORAL therapy

- cognitive restructuring, psychoeducation, breath retraining and muscle relaxation, teaching of self monitoring for symptoms, and vivo (real life) exposure to feared objects/situations

Examples of Maladaptive grieving

- complicated grieving - disenfranchised grieiving - public tragedies - Chronic Grief- - Delayed Grief - Exaggerated Grief

Guardianship

- court ordered relationship for acting on behalf of a person who is determined to be incompetent (adults)

what are some long term effects of sexual assault?

- depression - suicide - anxiety or fear - low self esteem - sexual dysfunction - somatic complaints

Directive to Physician

- doctor becomes surrogate to make decisions for patient when no family is around; especially if terminally ill

what is the benefit of BuSpar?

- does not cause dependence

Gerald Caplan (1964) crisis theory

- expanded crisis theory to all traumatic events and outlined crisis intervention strategies - his theory is grounded in the concept of homeostasis and returning individual to state of equilibrium - created the 4 phase crisis theory

MAIN treatment priority for eating disorder pt?

- getting them to eat - maintaining hydration

Lindemann (1940's) crisis theory

- grief interventions and brief therapy. Proposed that interventions could eliminate and decrease potential serious personality disorganization in the immediate aftermath of the crisis.

what would differentiate a diagnosis of major depression from bipolar disorder?

- if pt. has had manic episodes, lack of sleep, delusions, etc. —may be bipolar instead of depressive disorder

When are sexual assault survivors considered to be recovered?

- if they are relatively free of any signs or symptoms of acute stress disorder and PTSD. - Feel safe expressing their feeling - Lifestyle returns to normal - `Cessation of nightmares

how soon do symptoms develop in acute stress disorder?

- immediately after the event

other risk factors for schizophrenia:

- infection during pregnancy - stress, childhood sexual abuse, exposure to social adversity, migration to or growing up in a foreign culture, and exposure to psychological trauma or social defeat - toxins (the solvent tetrachlorothylene used in dry cleaning, to line water pipes, and sometimes found in drinking water)

Early Stress-Response Theories - Walter Cannon

- investigated the sympathetic nervous system in animals in response to stressors. He discovered the: - Fight (aggression) -or-flight (withdrawal) response - Body prepares for situation that individual perceives as threat to survival - Results in increased BP, HR, RR, and CO. - New research indicates that men and women have different neural responses to stress

advanced practice interventions for anxiety: BEHAVIORAL THERAPY

- involve teaching and physical practice of activities to decrease anxious or avoidant behavior - Modeling - Systematic desensitization - Flooding - Response prevention - Thought stopping

Anergia (Negative Symptom)

- lack of energy

avolition (negative symptom)

- lack of goals

Anhedonia (negative symptom)

- lack of pleasure

Anosognosia (negative symptom)

- lack of realizing you have a problem

How will sexually abused children present themselves?

- may be developmentally inappropriate - may speak/use words that are beyond their normal developmental stage

If depressive episode comes on due to maturational or situational life event...?

- may be on meds/in continuation phase for less time

If depressive episode comes on abrupt out of nowhere...?

- may be on meds/in maintenance phase longer

KEY NURSING DUTIES for delirium?

- monitor and observe! - Give them environmental aids, know baseline A&O, and know past cognitive impairments - Interact for only short concrete periods of time; family/familiarity may be helpful in short doses

what are the effects of incest on a victim?

- negative self image - depression - eating disorders - self destructive behavior - substance misuse

dissociative disorders

- occur after significant adverse experiences/traumas, and individuals response to stress with severe interruption of consciousness. Unconscious defense mechanism. - Protects individual against overwhelming anxiety through emotional separation

Stage 3. Exhaustion

- occurs when attempts to resist the stressor prove futile - resources depleted - stress may become chronic

Long-Acting Injectable Antipsychotics

- only need to be administered every 2-4 weeks

Beck (1976) theory on anger states?

- other threats such as values, beliefs, and moral code could also lead to anger

what are some examples of SSRIs

- paroxetine (Paxil) - fluoxetine (Prozac) - escitalopram (Lexapro) - fluvoxamine (Luvox) - sertraline (Zoloft)

Bereavement

- period of grieving following a death (more of a time period)

Durable Power of Attorney for Health Care

- person who makes decisions for family member

Living Will

- personal statement of how/when one wishes to die - may or may not be carried out

assessment for delirium?

- physical needs - mood and physical behaviors

what are the symptoms of dissociative disorders?

- positive : flashbacks - negative : memory problems, deficits in ability to control or sense different parts of the body

how is Adjustment disorder characterized?

- precipitated by a stressful event that is not considered traumatic. Responses to event include depression, anxiety, and conduct disturbances. Symptoms include guilt, depression, anxiety, anger, social withdrawal, work/academic inhibition.

onset of schizophrenia in children and males vs. females

- prevalence of onset in children is 1 in 40,000 - onset in males is between ages 15 and 25 and associated with poorer functioning and more structural abnormality in the brain - onset in women is 25-35 years old, who tend to have a better prognosis and experience less structural changes in the brain

SSRIs to treat major depression

- safer in regards to negative side effects/suicide/overdose Celexa, Prozac, Luvox, Paxil, Zoloft, Lexapro

Sigmund Freuds (1930) theory on anger states?

- the conflict between sexual needs and societal norms was the source of mankinds dissatisfaction, aggression, hostility, and violence.

Stage 1. Alarm

- the initial, brief and adaptive response (fight or flight) to the stressor

Menninger (2007) theory on anger states

- the struggle for control over our lives is a fundamental drive in every person. - if that control is threatened, we experience trauma, therefore instilling anger, aggression, and violence.

how do 2nd gen antipsychotics work?

- they antagonize dopamine receptors as well as bind to serotonin receptors

Mourning

- things people do to cope with grief (rituals; wakes, funerals; culture or family bound)

antianxiety drugs Pharmacological interventions for anxiety

- used to treat somatic and psychological symptoms of anxiety disorders. - Benzodiazepines most commonly used - (BuSpar) alternative antianxiety med that does not cause dependence but takes 2-4 weeks to work

Exceptions to patient confidentiality

-Duty to warn and protect third parties -Child and elder abuse reporting statutes

comorbidities of anger and aggression

-Posttraumatic stress disorder (PTSD) - Hypertension, myocardial infarction, ischemic strokes, metabolic syndrome - Substance abuse disorders Coexists with: - Depression - Anxiety - Psychosis - Personality disorders - Substance abuse disorders

How to rest patient POST ECT?

-REST ON SIDE

Western Tradition values

-autonomy -independence -self-reliance

Early Stress-Response Theories - Walter Cannon (1871-1945)

-investigated the sympathetic nervous system in animals in response to stressors. He discovered the: -Fight (aggression) -or-flight (withdrawal) response -Body prepares for situation that individual perceives as threat to survival -Results in increased BP, HR, RR, and CO.

CH 11 1. Which statement demonstrates a well structured attempt at limit setting? 1. "Hitting me when you are angry is unacceptable." 2. "I expect you to behave yourself during dinner." 3. ""Come here, right now!" 4. "Good boys dont bite."

1. "Hitting me when you are angry is unacceptable."

CH 11 what assessment question should the nurse ask when attempting to determine a teenager's mental health resilience? SELECT ALL THAT APPLY. 1. "How did you cope when your father deployed with the army for a year in Iraq?" 2. "Who did you go to for advice while your father was away for a year in Iraq?" 3. "How do you feel about talking to a mental health counselor?" 4. "Where do you see yourself in 10 years?" 5. "Do you like the school you go to?"

1. "How did you cope when your father deployed with the army for a year in Iraq?" 2. "Who did you go to for advice while your father was away for a year in Iraq?" 4. "Where do you see yourself in 10 years?"

A child diagnosed with attention deficit hyperactivity disorder (ADHD) is reprimanded for taking the nurse's pen without asking first. He responds by shouting, "You don't like me! You won't let me have anything, even a pen!" The nurse is most therapeutic when responding with which statement? 1. "I do like you, but I don't like it when you grab my pen." 2. "Liking you has nothing to do with whether I will loan you my pen." 3. "It sounds as though you are feeling helpless and insecure." 4. "You must ask for permission before taking someone else's things."

1. "I do like you, but I don't like it when you grab my pen." - This reply shows positive regard for the child while describing the behavior as undesirable. Feedback such as this helps the child feel accepted while making her aware of the effect her behavior has on others. None of the other options provide the necessary degree of positive regard

2. which statement made by the patient demonstrates an understanding of the effective use of newly prescribed lithium to manage bipolar mania? SELECT ALL THAT APPLY. 1. "I have to keep reminding myself to constantly drink six 12-ounce glasses of fluid everyday." 2. "I discussed the diuretic my cardiologist prescribed with my psychiatric care provider." 3. "Lithium may help me lose the few extra pounds i tend to carry around." 4. "I take my lithium on an empty stomach to help with absorption." 5. "Ive already made arrangements for my monthly lab work."

1. "I have to keep reminding myself to constantly drink six 12-ounce glasses of fluid everyday." 2. "I discussed the diuretic my cardiologist prescribed with my psychiatric care provider." 5. "Ive already made arrangements for my monthly lab work."

A depressed, socially withdrawn client tells the nurse, "There is no sense in trying. I am never able to do anything right!" The nurse can best address this cognitive distortion with which response? 1. "Let's look at what you just said, that you can 'never do anything right.'" 2. "Tell me what things you think you are not able to do correctly." 3. "Is this part of the reason you think no one likes you?" 4. "That is the most unrealistic thing I have ever heard."

1. "Let's look at what you just said, that you can 'never do anything right.'" - Cognitive distortions can be refuted by examining them, but to examine them the nurse must gain the client's willingness to participate. None of the other options examines the underlying cause of the feeling

Which statement made by a patient demonstrates a healthy degree of resilience? SELECT ALL THAT APPLY 1. "i try to remember not to take other peoples bad moods personally." 2. "i know that if i get really mad ill end up being depressed." 3. "i really feel that sometimes bad things are meant to happen." 4. "ive learned to calm down before trying to defend my opinions." 5. "i know that discussing issues with my boss would help me get my point across."

1. "i try to remember not to take other peoples bad moods personally." 4. "ive learned to calm down before trying to defend my opinions." 5. "i know that discussing issues with my boss would help me get my point across."

What are some risk factors that may predict violent behavior?

1. A history of violence is the single best predictor of future violence. 2. Patients who are delusional, hyperactive, impulsive, or predisposed to irritability are at higher risk for violence. 3. Assess the risk for violence: Does the patient have a wish or intent to harm? Does the patient have a plan? Does the patient have means available to carry out the plan? Does the patient have demographic risk factors (i.e., male gender, age 14 to 24 years, low socioeconomic status, inadequate support system, prison time)? 4. Aggression by patients occurs most often in the context of "limit setting" by the nurse. 5. Patients with a history of limited coping skills, including lack of assertiveness or use of intimidation, are at higher risk of using violence. 6. Assess self for personal triggers and responses likely to escalate patient violence, including patient characteristics or situations that trigger impatience, irritation, or defensiveness. 7. Assess personal sense of competence when in any situation of potential conflict; consider asking for the assistance of another staff member.

The family of a child diagnosed with attention deficit hyperactivity disorder (ADHD), inattentive type, is told the evaluation of their child's care will focus on symptom patterns and severity. What is the focus of child's evaluation? Select all that apply. 1. Academic performance 2. Activities of daily living 3. Physical growth 4. Social relationships 5. Personal perception

1. Academic performance 2. Activities of daily living 4. Social relationships 5. Personal perception - For the family and child with ADHD, evaluation will focus on the symptom patterns and severity. For those with ADHD, inattentive type, the focus of evaluation will be academic performance, activities of daily living, social relationships, and personal perception. For those with ADHD, hyperactive-impulsive type or combined type, the focus will be on both academic and behavioral responses.

Four Tasks of Mourning

1. Accept the reality of the loss. 2. Process the pain of grief while caring for the self. 3. Adjust to a world without the deceased. 4. Find an enduring connection with the deceased in the midst of embarking on a new life.

he physician mentions to the nurse that a client who is about to be admitted has "sundowning." The nurse can expect to assess for which nightly behavior? 1. Agitation 2. Lethargy 3. Depression 4. Mania

1. Agitation - Sundowning involves increased disorientation and agitation occurring at night. None of the other options are associated with sundowning

3 stages of GAS?

1. Alarm 2. Resistance 3. Exhaustion

2 main effects of crack and cocaine on body?

1. Anesthetic 2. Stimulant

5 principles of bioethics

1. Beneficence: The duty to promote good 2. Autonomy: Respecting the rights of others to make their own decisions 3. Justice: Distribute resources or care equally 4. Fidelity (nonmaleficence): Maintaining loyalty and commitment; doing no wrong to a patient 5. Veracity: One's duty to always communicate truthfully

CH 11 in pediatric mental health there is a lack of sufficient numbers of community based resources and providers, and there are long waiting lists for services. This has resulted in: SELECT ALL THAT APPLY 1. Children of color and poor economic conditions being underserved 2. Increased stress in the family unit 3. Markedly increased funding 4. Premature termination of services

1. Children of color and poor economic conditions being underserved 2. Increased stress in the family unit 4. Premature termination of services

5 constructs of cultural competence

1. Cultural awareness 2. Cultural knowledge 3. Cultural encounters 4. Cultural skill 5. Cultural desire

Criteria to justify admissions for inpatient care

1. Danger to self or others or 2. Unable to fulfill basic needs

The family members of a client with early-stage Alzheimer's disease cannot provide adequate supervision for the client. What would be a reasonable alternative for the nurse to explore with them to meet their current needs? 1. Day care 2. Acute care hospitalization 3. Long-term institutionalization 4. Group home residency

1. Day care - Day care Day care is a good option for clients with early-stage Alzheimer's disease. It provides supervision, a protected environment, and supportive interactions. The other options may be considered as the client moves into the advances stages of the disease disorder.

When preparing educational materials for the family of a client diagnosed with progressive dementia, the nurse should include information related to which local resourses? Select all that apply. 1. Day care centers 2. Legal professionals 3. Home health services 4. Family support groups 5. Professional counseling

1. Day care centers 3. Home health services 4. Family support groups 5. Professional counseling

A family member reports that the client had been oriented and able to carry on a logical conversation last evening, but this morning is confused and disoriented. The nurse can suspect that the client is displaying symptoms associated with which cognitive disorder? 1. Delirium 2. Dementia 3. Amnesic disorder 4. Selective inattention

1. Delirium - Delirium is characterized by a disturbance of consciousness, a change in cognition (such as impaired attention span), and a fluctuating level of consciousness that develop over a short period of time. None of the other options share these characteristics

3 common signs of STIMULANT abuse

1. Dilation of the pupils 2. Dryness of the oronasal cavity 3. Excessive motor activity

Admission options for inpatient care

1. Direct admission 2. Hospital emergency department

Date rape drugs commonly produce ?

1. Disinhibition 2. Relaxation of voluntary muscles 3. Anterograde amnesia

5 elements to prove negligence

1. Duty 2. Breach of duty 3. Cause in fact - "If it were not for what this nurse did (or failed to do) would this injury have occurred?" 4. Proximate cause 5. Damages

The nurse is expected to perform an assessment of a client suspected to be in the earliest stage of Alzheimer's disease. What finding would be out of character for the client who truly has early stage Alzheimer's disease? Select all that apply. 1. Easily frustrated by cognitive losses 2. Charming behavior designed to hide memory deficit 3. Confabulation to compensate for forgotten information 4. Avoidance of questions by subject changing

1. Easily frustrated by cognitive losses - Frustration and anger are characteristics of the middle stage of Alzheimer's. During early-stage Alzheimer's disease the client is aware of memory impairment and may attempt to disguise it or cover it by being evasive or using confabulation. The remaining options are associated with the early stage of Alzheimer's disease

For survivors to return to their previous level of functioning they must:

1. Fully mourn their losses 2. Experience anger 3. Work through their terrifying fears - can take years to fully work it through

A client hospitalized with anorexia nervosa has a weight that is 65% of normal. For this client, what is a realistic short-term goal for the first week of hospitalization regarding the physical impact of his/her weight? 1. Gain a maximum of 3 lb. 2. Develop a pattern of normal eating behavior. 3. Discuss fears and feelings about gaining weight. 4. Verbalize awareness of the sensation of hunger.

1. Gain a maximum of 3 lb. - The critical outcome during hospitalization for anorexia nervosa is weight gain. A maximum of 3 pounds weekly is considered sufficient initially. Too-rapid weight gain can cause pulmonary edema. While all the remaining goals are appropriate, none have the physical focus that is the initial priority

Which behavior would be characteristic of a client during a manic episode? 1. Going rapidly from one activity to another 2. Taking frequent rest periods and naps during the day 3. Being unwilling to leave home to see other people 4. Watching others intently and talking little

1. Going rapidly from one activity to another - Hyperactivity and distractibility are basic to manic episodes. None of the other options demonstrate such characteristics.

What is the Cycle of Violence?

1. Honeymoon Phase - abuser is loving, brings gifts, says sorry and they will change - victim is trusting, hoping for change, wants to believe partner 2. Tension-building - abuser is edgy, has minor explosions, may become verbally abusive, slapping, minor hitting - victim feels tense and afraid "walking on eggshells", feels helpless and becomes compliant 3. serious battering phase - tension becomes unbearable, victim may provoke incident to get it over with, victim may try to cover up the injury or may look for help

3 Most important question to ask when assessing pt for substance abuse?

1. In the last year, have you ever drank or used drugs more than you meant to? 2. Have you felt you wanted or needed to cut down on your drinking or drug use in the last year? 3. When was your last drink? - must know! Need to know for detox, surgery, medication administrator, etc.

ADHD consists of an Inappropriate degree of what 3 characteristics?

1. Inattention 2. Impulsiveness 3. Hyperactivity

Erickson's Psychosocial Stages of Development

1. Infancy 2. Early childhood 3. Late childhood 4. School age 5. Adolescence 6. Early adulthood 7. Middle Adulthood 8. Later Years

CH 22 A pt with a history of alcohol use disorder has been prescribed disulfiram (Antabuse). which physical effects support the suspicion that the pt has relapsed? SELECT ALL THAT APPLY. 1. Intense nausea 2. Diaphoresis 3. acute paranoia 4. confusion 5. dyspnea

1. Intense nausea 2. Diaphoresis 4. confusion

Main questions to ask when assessing child/adolescent?

1. Is there any Hx of mental illness in family? 2. is there any abuse?

Which signs and symptoms are associated with opioid withdrawal? 1. Lacrimation, rhinorrhea, dilated pupils, and muscle aches. 2. Illusions, disorientation, tachycardia, and tremors. 3. Fatigue, lethargy, sleepiness, and convulsions. 4. Synesthesia, depersonalization, and hallucinations.

1. Lacrimation, rhinorrhea, dilated pupils, and muscle aches. - Symptoms of opioid withdrawal resemble the "flu"; they include runny nose, tearing, diaphoresis, muscle aches, cramps, chills, and fever.

CH 11 5. Which factors tend to increase the difficulty of diagnosing young children who demonstrate behaviors associated with mental illness? SELECT ALL THAT APPLY. 1. Limited language skills 2. level of cognitive development 3. Level of emotional development 4. Parental denial that a problem exists 5. Severity of the typical mental illnesses observed in young children

1. Limited language skills 2. level of cognitive development 3. Level of emotional development

What is the first-line drug used to treat mania? 1. Lithium carbonate 2. Carbamazepine 3. Lamotrigine 4. Clonazepam

1. Lithium carbonate - Lithium, a mood stabilizer, is the first-line drug for use in treating bipolar disorder. The other options are prescribed to manage other related symptoms of bipolar disorder

outcome identification for anger and aggression?

1. Maintains self-control without supervision 2. Upholds contract to restrain aggressive behaviors

Five Properties of Defense Mechanisms

1. Manage conflict and affect - Protect people from anxiety - Maintain self-image by blocking feelings conflicts and memories 2. Automatic coping styles - Relatively unconscious 3. Discrete from one another 4. Often hallmarks of psychiatric syndromes, but reversible 5. Adaptive as well as pathological - healthy (adaptive) lower anxiety in acceptable ways - unhealthy (maladaptive) use of DM in excess - Use determined by: frequency, duration, intensity

what are the different levels of anxiety?

1. Mild 2. Moderate 3. Severe 4. Panic

When providing care for a client diagnosed with borderline personality disorder, the nurse will need to consider strategies for dealing with which of the client's classic characteristics? 1. Mood shifts, impulsivity, and splitting 2. Grief, anger, and social isolation 3. Altered sensory perceptions and suspicion 4. Perfectionism and preoccupation with detail

1. Mood shifts, impulsivity, and splitting - Borderline personality disorder has the central characteristic of instability in affect, identity, and relationships. Borderline individuals desperately seek relationships to avoid feeling abandoned, but they often drive others away with excessive demands, impulsive behavior, or uncontrolled anger. Their frequent use of the defense of splitting strains personal relationships and creates turmoil in health care settings. The remaining options suggest characteristics not associated with this disorder.

Four Common Problems in the Nurse-Patient Relationship

1. Nurse needs to be needed 2. Nurse sets unrealistic goals for patient 3. Nurse has difficulty dealing with the issue of suicide 4. Nurse has difficulty terminating the nurse-patient relationship

Which statement about antidepressant medications, in general, can serve as a basis for client and family teaching? 1. Onset of action is from 1 to 3 weeks or longer. 2. They tend to be more effective for men. 3. Recent memory impairment is commonly observed. 4. They often cause the client to have diurnal variation.

1. Onset of action is from 1 to 3 weeks or longer. - A drawback of antidepressant drugs is that improvement in mood may take 1 to 3 weeks or longer. None of the other options provide correct information regarding antidepressant medications

Which statement is true of the eating disorder referred to as bulimia? 1. Patients with bulimia often appear at a normal weight. 2. Patients with bulimia binge eat but do not engage in compensatory measures. 3. Patients with bulimia severely restrict their food intake. 4. One sign of bulimia is lanugo.

1. Patients with bulimia often appear at a normal weight. - Patients with bulimia are often at or close to ideal body weight and do not appear physically ill. The other options do not refer to bulimia but rather refer to signs of binge eating disorder and anorexia nervosa

according to Maslow's Hierarchy of needs, the most basic needs category for nurses to address is: 1. Physiological 2. Safety 3. Love and belonging 4. Self-actualization

1. Physiological

TWO key signs of conduct disorder?

1. Pyromania - Maladaptive coping mechanism due to unmet needs and poor social skills 2. Kleptomania - May be related to addictive disorders due to satisfying a compulsion

Which factor can reduce the vulnerability of a child to etiological influences predisposing to the development of psychopathology? 1. Resilience 2. Malnutrition 3. Child abuse 4. Having a depressed parent

1. Resilience - Resilience refers to developing and using certain characteristics that help a child to handle the stresses of a difficult childhood without developing mental problems. Resilient children can adapt to changes in the environment, form nurturing relationships with adults other than their parents, distance themselves from the emotional chaos of the family, and have social intelligence and the ability to use problem-solving

A young woman reports that although she has no memory of the event, she believes that she was raped. This raises suspicion that she unknowingly ingested what substance? Select all that apply. 1. Rohypnol 2. Gamma-hydroxybutyrate (GHB) 3. ReVia 4. Clonidine 5. Ayahuasca

1. Rohypnol 2. Gamma-hydroxybutyrate (GHB) - 1. The drugs most frequently used to facilitate a sexual assault (rape) are flunitrazepam (Rohypnol, "roofies"), a fast-acting benzodiazepine, and gamma-hydroxybutyrate (GHB) and its congeners. These drugs are odorless, tasteless, and colorless; mix easily with drinks; and can render a person unconscious in a matter of minutes. Perpetrators use these drugs because they rapidly produce disinhibition and relaxation of voluntary muscles; they also cause the victim to have lasting anterograde amnesia for events that occur. The description is not associated with any of the other suggested drugs - 2. The drugs most frequently used to facilitate a sexual assault (rape) are flunitrazepam (Rohypnol, "roofies"), a fast-acting benzodiazepine, and gamma-hydroxybutyrate (GHB) and its congeners. These drugs are odorless, tasteless, and colorless; mix easily with drinks; and can render a person unconscious in a matter of minutes. Perpetrators use these drugs because they rapidly produce disinhibition and relaxation of voluntary muscles; they also cause the victim to have lasting anterograde amnesia for events that occur. The description is not associated with any of the other suggested drugs.

SAD PERSONS scale

1. Sex 2. Age 3. Depression 4. Previous Attempts 5. Ethanol or Other Drugs 6. Rational Thinking Loss 7. Social Supports Lacking or recent loss 8. Organized Plan 9. No Spouse 10. Sickness

A 5-year-old who consistently omits the sound for 'r' and 's' when speaking is demonstrating which type of disorder? 1. Speech 2. Language 3. Social communication 4. Specific learning

1. Speech - Speech disorders are marked by problems in making sounds. Children may have trouble making certain sounds, or they may distort, add, or omit sounds. Such patterns are not associated with any of the other

Which term is broader? 1. Spirituality 2. Religion 3. Not sure

1. Spirituality - Spirituality addresses universal human questions and needs. Spirituality is cognitive, experiential, and behavioral. In contrast, religion is an external system that includes beliefs, patterns of worship, and symbols. Although religion is often concerned with spirituality, religious groups are social entities and are often characterized by other nonspiritual goals (cultural, economic, political, social).

The nurse working with clients diagnosed with eating disorders can help families develop effective coping mechanisms by implementing which intervention? 1. Teaching the family about the disorder and the client's behaviors 2. Stressing the need to suppress overt conflict within the family 3. Urging the family to demonstrate greater caring for the client 4. Encouraging the family to use their usual social behaviors at meals

1. Teaching the family about the disorder and the client's behaviors - Families need information about specific eating disorders and the behaviors often seen in clients with these disorders. This information can serve as a basis for additional learning about how to support the family member. While the other options may be appropriate for specific client families, they are not as fundamental as the correct option

When a nurse assesses the style of behavior a child habitually uses to cope with the demands and expectations of the environment, he or she is assessing characteristic? 1. Temperament 2. Resilience 3. Vulnerability 4. Cultural assimilation

1. Temperament - Temperament is the behavior the child habitually uses to cope with the environment. It is a constitutional factor thought to be genetically determined. It may be modified by the parent-child relationship. None of the other options would reflect this characteristic

An acute phase nursing intervention aimed at reducing hyperactivity is demonstrated by which intervention? 1. Writing in a diary 2. Exercising in the gym 3. Directing unit activities 4. Orienting a new client to the unit

1. Writing in a diary - Manic clients often respond well to the invitation to write. They will fill reams of paper. While writing they are less physically active. None of the remaining options presents this opportunity to reduce physical activity.

CH 14 which nursing intervention focuses on managing a common characteristic of major depressive disorder associated with the older population ? 1. conducting routine suicide screenings at a senior center 2. identifying depression as a natural, but treatable result of aging 3. identifying males as being at a greater risk for developing depression 4. stressing that most individuals experience just a single episode of major depression in a lifetime

1. conducting routine suicide screenings at a senior center

best practice guidelines for a person reporting sexual assault

1. head to toe physical assessment 2. detailed genital exam 3. evidence collection and preservation 4. documentation of physical findings 5. treatment, discharge planning, follow up care

CH 13 which interventions should the nurse implement when caring for a pt demonstrating manic behavior? SELECT ALL THAT APPLY. 1. monitor vital signs frequently 2. provide pt with frequent milkshakes and protein shakes 3. reduce the volume on the television and dim bright lights in the room 4. keep the pt distracted with group-oriented activites 5. use a firm but calm voice to give specific concise directions to the pt

1. monitor vital signs frequently 2. provide pt with frequent milkshakes and protein shakes 3. reduce the volume on the television and dim bright lights in the room 5. use a firm but calm voice to give specific concise directions to the pt

which nursing actions demonstrates cultural competence? SELECT ALL THAT APPLY 1. planning mealtime around the pt's prayer schedule 2. helping a pt to visit the hospital chaplain 3. researching foods that a lacto-ovo-vegetarian patient will eat 4. providing time for a pt's spiritual healer to visit 5. ordering standard meal trays to be delivered 3 times daily.

1. planning mealtime around the pt's prayer schedule 2. helping a pt to visit the hospital chaplain 3. researching foods that a lacto-ovo-vegetarian patient will eat 4. providing time for a pt's spiritual healer to visit

Disaster management continuum

1. preparedness 2. mitigation 3. response 4. recovery 5. evaluation

Evaluation for dissociative disorders

1. pt. safety maintained 2. reduced anxiety, pt. returns to normal functioning state 3. integration of fragmented memories 4. new coping strategies 5. better handle of stress

CH 22. which assessment data confirm the suspicion that the pt is experiencing opioid withdrawal? SELECT ALL THAT APPLY. 1. pupils are dilated 2. pulse rate is 62 beats/min 3. slow movements 4. extreme anxiety 5. sleepy

1. pupils are dilated 4. extreme anxiety

How much caffeine should you have per day?

100-300 mg/day

A new psychiatric technician mentions to the nurse, "Depression seems to be a disease of old people. All the depressed clients on the unit are older than 60 years." How should the nurse respond to this statement? 1. "That is a good observation. Depression does mostly strike people older than 50 years." 2. "Depression is seen in people of all ages, from childhood to old age." 3. "Depression is most often seen among the middle adult age group." 4. "The age of onset for most depressive episodes is given as 18 years."

2. "Depression is seen in people of all ages, from childhood to old age." - Depression can occur at any age. Children, adolescents, adults, and the elderly may all experience depression.

CH 11 3. Cognitive behavioral therapy is going well when a 12-year old pt in therapy reports to the nurse practitioner: 1. "I was so mad i wanted to hit my mother." 2. "I thought that everyone at school hated me. Thats not true. Most people like me and i have a friend named Todd." 3. "I forgot that you told me to breathe when i become angry." 4. "i scream as loud as i can when the train goes by the house."

2. "I thought that everyone at school hated me. Thats not true. Most people like me and i have a friend named Todd."

CH 14 Which statement would best show acceptance of a depressed, mute client? 1. "I will be spending time with you each day to try to improve your mood." 2. "I would like to sit with you for 15 minutes now and again this afternoon." 3. "Each day we will spend time together to talk about things that are bothering you." 4. "It is important for you to share your thoughts with someone who can help you evaluate your thinking."

2. "I would like to sit with you for 15 minutes now and again this afternoon." - Spending time with the client without making demands is a good way to show acceptance. While not inappropriate, the other options are less accepting

which question should the nurse ask when assessing for what Sullivan's Interpersonal Theory identifies as the most painful human condition? 1. "is self-esteem important to you?" 2. "do you think of yourself as being lonely?" 3. "what do you do to manage your anxiety?" 4. "have you ever been diagnosed with depression?"

2. "do you think of yourself as being lonely?"

CH 14 Which assessment question asked by the nurse demonstrates an understanding of comorbid mental health conditions associated with major depressive disorder? SELECT ALL THAT APPLY. 1. "do rules apply to you?" 2. "what do you do to manage anxiety?" 3. "do you have a hx of disordered eating?" 4. "do you think you drink too much?" 5. "have you ever been arrested for committing a crime?"

2. "what do you do to manage anxiety?" 3. "do you have a hx of disordered eating?" 4. "do you think you drink too much?"

A client diagnosed with Alzheimer's disease looks confused and cannot recall many common household objects by name, such as a pencil or glass. The nurse should document this loss of function using which term? 1. Apraxia 2. Agnosia 3. Aphasia 4. Anhedonia

2. Agnosia - Agnosia is a loss of the ability to recognize familiar objects. The loss is not associated with any of the other options.

A 16-year-old patient being treated for anorexia, has been prescribed medication to reduce compulsive behaviors regarding food now that ideal weight has been reached. Which class of medication is prescribed for this specific issue associated with eating disorders? 1. Mood stabilizers 2. Antidepressants 3. Anxiolytics 4. Atypical antipsychotics

2. Antidepressants - The antidepressant fluoxetine (Prozac, an SSRI) has proven useful in reducing obsessive-compulsive behavior after the patient has reached a maintenance weight. Anxiolytics would be prescribed for anxiety. Atypical antipsychotic agents may be helpful in improving mood and decreasing obsessional behaviors and resistance to weight gain. Mood stabilizers are not specifically used in treatment of eating disorders.

CH 13 Tatiana has been hospitalized for an acute manic episode. on admission the nurse suspects lithium toxicity. what assessment findings would indicate the nurses suspicion as correct? 1. Shortness of breath, GI distress, chronic cough 2. Ataxia, severe hypotension, large volume of dilute urine 3. GI distress, thirst, nystagmus 4. electroencephalographic changes, chest pain, dizziness

2. Ataxia, severe hypotension

A 7-year-old, who is described as impulsive and hyperactive, tells the nurse, "I am a dummy, because I don't pay attention, and I can't read like the other kids." The nurse notes that these behaviors are most consistent with which diagnosis? 1. Attention deficit disorder 2. Attention deficit hyperactivity disorder 3. Autism 4. Conduct disorder

2. Attention deficit hyperactivity disorder - The data are most consistent with attention deficit hyperactivity disorder (ADHD) as described in the DSM-5. The other options present with characteristics and behaviors that differ from those in the scenario.

Which social behavior is often a result of a child having been exposed to some form of abuse? 1. Speech disorders 2. Bullying others 3. Eating disorders 4. Delayed motor skills

2. Bullying others - Children who have experienced abuse are at risk for identifying with their aggressor and may act out, bully others, become abusers, or develop dysfunctional interpersonal relationships in adulthood. None of the remaining options are as directly associated with abuse as bullying

a gulf war veteran has been homeless since being discharged from military service. He is now diagnosed with schizophrenia. The nurse practitioner recognizes that assertive community treatment (ACT) is a good option for this pt since ACT provides: 1. Psychiatric home-care 2. Care for hard-to-engage, seriously ill patients 3. Outpatient community mental health center care 4. A comprehensive emergency service model

2. Care for hard-to-engage, seriously ill patients

The nurse can expect a client demonstrating typical manic behavior to be attired in clothing that includes with characteristics? 1. Dark colored and modest 2. Colorful and outlandish 3. Compulsively neat and clean 4. Ill-fitted and ragged

2. Colorful and outlandish - Manic clients often manage to dress and apply makeup in ways that create a colorful, even bizarre, appearance. None of the remaining options meet that criteria

A client diagnosed with bulimia nervosa uses enemas and laxatives to purge to maintain weight. What is the likely physiological outcome of this practice? 1. Increase in the red blood cell count 2. Disruption of the fluid and electrolyte balance 3. Elevated serum potassium level 4. Elevated serum sodium level

2. Disruption of the fluid and electrolyte balance ~ Disruption of the fluid and electrolyte balance is usually the result of excessive use of enemas and laxatives. There would be a decrease in potassium and sodium levels while the concentration of but not actual red cell count would be affected

A client reveals that she induces vomiting as often as a dozen times a day. The nurse would expect assessment findings to support which electrolyte imbalance? 1. Hypernatremia 2. Hypokalemia 3. Hypercalcemia 4. Hypolipidemia

2. Hypokalemia ~ Vomiting causes loss of potassium, leading to hypokalemia. Vomiting is not the trigger for any of the other options presented.

When a delirious client insists that a vacuum hose is a large, poisonous snake, the nurse recognizes that this client is experiencing what characteristic symptom? 1. Hallucinations 2. Illusion 3. Hypervigilant 4. Agnosia

2. Illusion - Illusions are errors in the perception of a sensory stimulus. None of the other options are associated with this form of misperception.

When educating a client diagnosed with bulimia nervosa about the medication fluoxetine, the nurse should include what information about this medication? 1. It will reduce the need for cognitive therapy. 2. It will be prescribed at a higher than typical dose. 3. There are a variety of medications to prescribe if fluoxetine proves to be ineffective. 4. Long-term management of symptoms is best achieved with tricyclic antidepressants.

2. It will be prescribed at a higher than typical dose. - Research has shown that antidepressant medication together with cognitive-behavioral therapy brings about improvement in bulimic symptoms. Fluoxetine (Prozac), an Selective serotonin reuptake inhibitors (SSRI) antidepressant, has FDA approval for acute and maintenance treatment of bulimia nervosa in adult patients. When fluoxetine is used for bulimia, it is typically at a higher dose than is used for depression. Although no other drugs have FDA approval for this disorder, tricyclic antidepressants helped reduce binge eating and vomiting over short terms.

A nurse assesses a newly admitted client diagnosed with Alzheimer's disease and a urinary tract infection. The nurse asks the client's sibling for information about the home environment, ADLs, and medications. What type of information source is the sibling? 1. Primary 2. Secondary 3. Private 4. Informed

2. Secondary - Secondary sources are valuable when caring for a client experiencing psychosis, muteness, agitation, or catatonia. Such secondary sources include members of the family, friends, neighbors, police, health care workers, and medical records.

What is the major reason for the hospitalization of a depressed patient? 1. Inability to go to work 2. Suicidal ideation 3. Loss of appetite 4. Psychomotor agitation

2. Suicidal ideation - Suicidal thoughts are a major reason for hospitalization for patients with major depression. It is imperative to intervene with such patients to keep them safe from self-harm. The other options describe symptoms of major depression but aren't by themselves the major reason for hospitalization.

When the nurse asks whether a client is having any thoughts of suicide, the client becomes angry and defensive, shouting, "I'm sick of you people! Are you ever do is ask me the same question over and over. Get out of here!" The nurse's response is based on what fact concerning hostility? 1. The client is getting better and is able to be assertive. 2. The client may be at high risk for self-harm. 3. The client is probably experiencing transference. 4. The client may be angry at someone else and projecting that anger to staff.

2. The client may be at high risk for self-harm. - Overt hostility is highly correlated with suicide; therefore the patient may be considered high risk, and appropriate precautions should be taken. The other responses are incorrect with no evidence to support them.

Which intervention would be least useful for accurate assessment of the weight of a client diagnosed with anorexia nervosa? 1. Weigh 2 times daily first week, then three times weekly. 2. Weigh fully clothed before breakfast. 3. Do not reweigh client when client requests. 4. Permit no oral intake before weighing.

2. Weigh fully clothed before breakfast. - Clients should be weighed daily first week, then three times weekly wearing only bra and panties or underwear before ingesting any food or fluids in the morning. Reweighing is not a request that should be afforded to the client.

Which worldview would the nurse anticipate from a client who says, "It is important to save enough money to take care of yourself in your old age. We should not rely on anyone else to take care of us." 1. Eastern (balance) 2. Western (science) 3. Indigenous (harmony)

2. Western (science) In the Western tradition, one's identity is found in one's individuality, which inspires the valuing of autonomy, independence, and self-reliance. Success in life is obtained by preparing for the future.

CH 13 substance abuse is often present in people diagnosed with bipolar disorder. Laura, a 28-yr old with a diagnosis of bipolar disorder, drinks alcohol instead of taking her prescribed medications. the nurse caring for this pt recognizes that: 1. anxiety may be present 2. alcohol ingestion is a form of self medication 3. the pt. is lacking a significant amount of neurotransmitters. 4. the pt. is using alcohol bc she is depressed.

2. alcohol ingestion is a form of self medication

A client prescribed a monamine oxidase inhibitor (MOA) has a pass to go out to lunch. Given a choice of the following entrees, the client can safely eat 1. avocado salad plate. 2. fruit and cottage cheese plate. 3. kielbasa and sauerkraut. 4. liver and onion sandwich.

2. fruit and cottage cheese plate. - Fruit and cottage cheese do not contain tyramine. Avocados, fermented food such as sauerkraut, processed meat, and organ meat, contain tyramine. Monoamine oxidase inhibitors inhibit the breakdown of tyramine, which can lead to high blood pressure, a hypertensive crisis, and eventually a cerebrovascular accident. This information makes the other options incorrect.

A child diagnosed with autism will demonstrate impaired development in which area? 1. Adhering to routines 2. playing with other children 3. swallowing and chewing 4. eye-hand coordination

2. playing with other children ~ Autism affects the normal development of the brain in social interaction and communication skills. Symptoms associated with autism spectrum disorders include significant deficits in social relatedness, including communication, nonverbal behavior, and age-appropriate interaction. Other behaviors include stereotypical repetitive speech, obsessive focus on specific objects, over adherence to routines or rituals, hyper- or hypo-reactivity to sensory input, and extreme resistance to change. None of the other options are characteristically associated with autism.

Which nursing diagnosis should be considered for a child with attention deficit hyperactivity disorder ADHD? 1. Anxiety 2. risk for injury 3. defensive coping 4. impaired verbal communication Submit

2. risk for injury - The child's marked hyperactivity puts him or her at risk for injury from falls, bumping into objects, impulsively operating equipment, pulling heavy objects off shelves, and so forth

Length of seizure during ECT?

20 seconds - 1 minute - ideally want 40-45 sec / seizure

New Freedom Commission on Mental Health

2003 - Release of recommendations for mental health care in America. Called for a streamlined system. Advocated for Early diagnosis and treatment, New expectation for principles of recovery, Increased assistance in helping people find housing & work.

When will detox tremens (DT) become evident?

24-48 hours after last drink

A client prescribed fluoxetine demonstrates an understanding of the medication teaching when making which statement? 1. "I will make sure to get plenty of sunshine and not use sunscreen to avoid a skin reaction." 2. "I will not take any over-the-counter medication while on the fluoxetine." 3. "I will report any symptoms of high fever, fast heartbeat, or abdominal pain to my provider right away." 4. "I will report increased thirst and urination to my provider."

3. "I will report any symptoms of high fever, fast heartbeat, or abdominal pain to my provider right away." - This describes symptoms of serotonin syndrome, a life-threatening complication of SRRI medication. The other options are incorrect because the patient should be wearing sunscreen to avoid sunburn, may take over-the-counter medications if sanctioned by the provider, and would not have been educated to report increased thirst and urination as a side effect of fluoxetine.

CH 13 A 33 year old female diagnosed with Bipolar I disorder has been functioning well on lithium for 11 months. at her most recent checkup, the psychiatric nurse practitioner states "You are ready to enter the maintenance therapy stage, so at this time i am going to adjust your dosage by prescribing: 1. A higher dosage 2. Once a week dosing 3. A lower dosage 4. A different drug

3. A lower dosage

besides antianxiety agents, which classification of drugs is also commonly given to treat anxiety and anxiety disorders? 1. Antipsychotics 2. Mood stabilizers 3. Antidepressants 4. Cholinesterase inhibitors

3. Antidepressants

What is the priority nursing intervention for a client diagnosed with borderline personality disorder? 1. Protect other clients from manipulation. 2. Respect the client's need for attention. 3. Assess for suicidal and self-mutilating behaviors. 4. Provide clear, consistent limits and boundaries.

3. Assess for suicidal and self-mutilating behaviors. - One of the primary nursing guidelines/interventions for clients with a personality disorder is to assess for suicidal and self-mutilating behaviors, especially during times of stress. While the other options may be appropriate, none have the priority of safety.

When a child demonstrates a temperament that prompts the mother to say, "She is just so different from me; I just can't seem to connect with her." The nurse should plan to provide which intervention? 1. Suggest that the child's father become her primary caregiver. 2. Encourage the mother to consider attending parenting classes. 3. Counsel the mother regarding ways to better bond with her child. 4. Educate the father regarding signs that the child is being physically abused.

3. Counsel the mother regarding ways to better bond with her child. - All people have temperaments, and the fit between the child and parent's temperament is critical to the child's development. The caregiver's role in shaping that relationship is of primary importance, and the nurse can intervene to teach parents ways to modify their behaviors to improve the interaction

What term is used to identify the condition demonstrated by a person who has numerous hypomanic and dysthymic episodes over a two-year period? 1. Bipolar II disorder. 2. Bipolar I disorder. 3. Cyclothymia. 4. Seasonal affective disorder.

3. Cyclothymia. - Cyclothymia refers to mood swings involving hypomania and dysthymia of 2 years duration. The mood swings are not severe enough to prompt hospitalization. None of the other options meet that criteria.

CH 13 Ted, a former executive is now unemployed due to manic episodes at work. He was diagnosed with bipolar I 8 years ago. Ted has a hx of IV drug abuse, which resulted in Hep C. He is taking his lithium exactly as scheduled, a fact that both teds wife and blood work confirm. to reduce teds mania the psychiatric nurse practitioner recommends: 1. Klonopin 2. Prozac 3. Electrocompulsive therapy 4. Latuda

3. Electrocompulsive therapy

Which term refers to individuals' belief that their cultural values and practices are correct and superior to those of others? 1. Assimilation 2. Enculturation 3. Ethnocentrism 4. Somaticization

3. Ethnocentrism - Ethnocentrism is the universal tendency of humans to think that their way of thinking and behaving is the only correct and natural way.

Which behavior is most indicative of a 4-year-old child diagnosed with Tourette's syndrome? 1. Difficulty in social relationships 2. Humming while performing activities that require concentration 3. Frequent eye blinking 4. Difficulty in completing tasks on time

3. Frequent eye blinking - Persistent motor or vocal ticking is characteristic of Tourette's syndrome. Dysfunctional social relationship is an inconclusive symptom, especially for a 4-year-old. Humming can be a normal response of a child at play. Ineffective time management is usually associated with a child who demonstrates ADHD, not Tourette's syndrome.

A depressed client tells the nurse, "There is no sense in trying. I am never able to do anything right!" The nurse should identify this cognitive distortion as what response? 1. Self-blame 2. Catatonia 3. Learned helplessness 4. Discounting positive attributes

3. Learned helplessness - Learned helplessness results in depression when the client feels no control over the outcome of a situation. None of the other options demonstrate these feelings.

When the clinician mentions that a client has anhedonia, the nurse can expect that the client will demonstrate what behavior? 1. Poor retention of recent events 2. A weight loss from anorexia 3. No pleasure from previously enjoyed activities 4. Difficulty with tasks requiring fine motor skills

3. No pleasure from previously enjoyed activities - Anhedonia is the only term that suggests the lack of ability to experience pleasure

A depressed client is noted to pace most of the time, pull at her clothes, and wring her hands. These behaviors are consistent with which term? 1. Senile dementia 2. Hypertensive crisis 3. Psychomotor agitation 4. Central serotonin syndrome

3. Psychomotor agitation - These behaviors describe the psychomotor agitation sometimes seen in clients with the agitated type of depression. None of the other options are associated so directly with these behaviors.

CH 11 Child protective services have removed 10-year-old Christopher from his parents home due to neglect. Christopher reveals to the nurse that he considers the woman next door his "nice" mom, that he loves school, and gets above average grades. The strongest explanation of this response is: 1. Temperament 2. Genetic factors 3. Resilience 4. Paradoxical effects of neglect

3. Resilience

Which diagnosis from the list below would be given priority for a client diagnosed with bulimia nervosa? 1. Disturbed body image 2. Chronic low self-esteem 3. Risk for injury: electrolyte imbalance 4. Ineffective coping: impulsive responses to problems

3. Risk for injury: electrolyte imbalance - The client who engages in purging and excessive use of laxatives and enemas is at risk for metabolic acidosis from bicarbonate loss. This electrolyte imbalance is potentially life threatening. While appropriate none of the other options are as likely to risk the client's life

Biological theorists suggest that the cause of eating disorders may be related to which factor? 1. Normal weight phobia 2. Body image disturbance 3. Serotonin imbalance 4. Dopamine excess

3. Serotonin imbalance - The selective serotonin reuptake inhibitors have been shown to improve the rate of weight gain and reduce the occurrence of relapse. None of the remaining options are currently supported by any biological theories

when considering facility admissions for mental healthcare, what characteristic is unique to a voluntary admission? 1. the pt poses no substantial threat to themselves or others 2. the pt has the right to seek legal counsel 3. a request in writing is required before admission 4. a mental illness has been previously diagnosed

3. a request in writing is required before admission

when considering the suggestions of Hildegard Peplau, which activity should the nurse actively engage in to ensure that the pt stays the focus of all therapeutic conversations? 1. assessing the pt for unexpressed concerns and fears 2. evaluating the possible need for additional training and education 3. reflecting on personal behaviors and personal needs 4. avoiding power struggles with the manipulative pt

3. reflecting on personal behaviors and personal needs

A patient has been voluntarily admitted to a mental health facility after an unsuccessful attempt to harm himself. which statement demonstrates a need to BETTER educate the pt on his pt's rights? 1. "i understand why i was restrained when i was out of control." 2. "you cant tell my boss about the suicide attempt without my permission." 3. "i have a right to know what all of you plan to do to me." 4. "i can hurt myself if i want to. its none of your business."

4. "i can hurt myself if i want to. its none of your business."

CH 14 Which response by a 15-year-old demonstrates a common symptom observed in pts diagnosed with major depressive disorder? 1. "I am so restless. i cant seem to sit still." 2. "i spend most of my time studying. i have to get into a good college." 3. "im not trying to diet, but ive lost about 5 lbs in the past 5 months." 4. "i go to sleep around 11 p.m. but im always up by 3 A.M. and cant go back to sleep."

4. "i go to sleep around 11 p.m. but im always up by 3 A.M. and cant go back to sleep."

The nurse caring for a client diagnosed with Alzheimer's disease can anticipate that the family will need information about which medication therapy? 1. Antihypertensives 2. Benzodiazepines 3. Immunosuppressants 4. Acetylcholinesterase inhibitors

4. Acetylcholinesterase inhibitors - Memory deficit is thought to be related to a lack of acetylcholine at the synaptic level. Acetylcholinesterase inhibitor drugs prevent the chemical that destroys acetylcholine from acting, thus leaving more available acetylcholine.

Dysthymia cannot be diagnosed unless it has existed for what period of time? 1. At least 3 months 2. At least 6 months 3. At least 1 year 4. At least 2 years

4. At least 2 years - Dysthymia is a chronic condition that by definition has to have existed for longer than 2 years. None of the other options present a sufficient time period.

which standardized rating scale will the nurse specifically include in the assessment of a newly admitted patient diagnosed with major depressive disorder? 1. Mini-mental state exam (MMSE) 2. Body altitude test 3. Global assessment of functioning scale (GAF) 4. Beck inventory

4. Beck inventory

Benzodiazepines are useful for treating alcohol withdrawal because they are associated with which action? 1. Blocking cortisol secretion 2. Increasing dopamine release 3. Decreasing serotonin availability 4. Exerting a calming effect

4. Exerting a calming effect - Benzodiazepines act by binding to α-aminobutyric acid receptor sites, producing a calming effect. Benzodiazepines are not associated with any of the other suggested actions.

A client prescribed a selective serotonin reuptake inhibitor mentions taking the medication along with the St. John's wort daily. The nurse should provide the client with what information regarding this practice? 1. Agreeing that this will help the client to remember the medications. 2. Caution the client to drink several glasses of water daily. 3. Suggest that the client also use a sun lamp daily. 4. Explain the high possibility of an adverse reaction.

4. Explain the high possibility of an adverse reaction. - Serotonin malignant syndrome is a possibility if St. John's wort is taken with other antidepressants. None of the other options are relevant to the situation.

Which subjective symptom should the nurse would expect to note during assessment of a client diagnosed with anorexia nervosa? 1. Lanugo 2. Hypotension 3. 25-lb weight loss 4. Fear of gaining weight

4. Fear of gaining weight ~ Fear of weight gain is the only subjective data listed, and it is universally true of clients diagnosed with anorexia nervosa

The nurse can determine that inpatient treatment for a client diagnosed with an eating disorder would be warranted when which assessment data is observed? 1. Weighs 10% below ideal body weight. 2. Has serum potassium level of 3 mEq/L or greater. 3. Has a heart rate less than 60 beats/min. 4. Has systolic blood pressure less than 70 mm Hg.

4. Has systolic blood pressure less than 70 mm Hg. - Systolic blood pressure of less than 70 mm Hg is one of the established criteria signaling the need for hospitalization of a client with anorexia nervosa. It suggests severe cardiovascular compromise. None of the remaining options represent data aligned with the criteria for hospitalization.

CH 18 A client who is 16 years old, 5 foot, 3 inches tall, and weighs 80 pounds eats one tiny meal daily and engages in a rigorous exercise program. Which nursing diagnosis addresses this assessment data? 1. Death anxiety 2. Ineffective denial 3. Disturbed sensory perception 4. Imbalanced nutrition: less than body requirements

4. Imbalanced nutrition: less than body requirements ~ A body weight of 80 pounds for a 16-year-old who is 5 foot, 3 inches tall is ample evidence of this diagnosis. There is no support in the data as presented to justify any of the other nursing diagnoses.

In which part of the nursing care plan would the nurse expect to find this statement: Offer snacks and finger foods frequently. 1. Assessment 2. Diagnosis 3. Planning and outcomes identification 4. Intervention 5. Evaluation

4. Intervention

When a client experiences four or more mood episodes in a 12-month period, which term is used to describe this behavior? 1. Dyssynchronous 2. Incongruent 3. Cyclothymic 4. Rapid cycling

4. Rapid cycling - Rapid cycling implies four or more mood episodes in a 12-month period, as well as more severe symptomatology. None of the other options are associated with this characteristic behavior.

CH 11 2. which activity is most appropriate for a child with ADHD? 1. Reading an adventure novel 2. Monopoly 3. Checkers 4. Tennis

4. Tennis

When the wife of a manic client asks about genetic transmission of bipolar disorder, the nurse's answer should be predicated on which information? 1. No research exists to suggest genetic transmission. 2. Much depends on the socioeconomic class of the individuals. 3. Highly creative people tend toward development of the disorder. 4. The rate of bipolar disorder is higher in relatives of people with bipolar disorder.

4. The rate of bipolar disorder is higher in relatives of people with bipolar disorder. - This understanding will allow the nurse to directly address the question. Responses based on the other statements would be tangential or untrue.

A 10-year-old who is frequently disruptive in the classroom begins to fidget and then moves on to disruptive behavior. What is the most appropriate initial technique for managing this sort of disruptive behavior? 1. Therapeutic holding 2. Seclusion 3. Quiet room 4. Touch control

4. Touch control - The appropriate adult can move closer to the child and place a hand on his/her arm or an arm around his/her shoulder for a calming effect when the fidgeting is first noted. The closeness signals the child to use self-control. It is the least restrictive treatment approach and should be tried initially; before any of the other options

Which event would an older client diagnosed with early stage Alzheimer's disease have greatest difficulty remembering? 1. His or her high school graduation 2. The births of his or her children 3. The story of a teenage escapade 4. What he or she ate for breakfast

4. What he or she ate for breakfast - Initially, recent memory is impaired, and remote memory remains intact.

CH 13 A male pt calls to tell the nurse that his monthly lithium level is 1.7 mEq/L. which nursing intervention will the nurse implement initially? 1. reinforce that the level is considered therapeutic 2. instruct the pt to hold the next dose of medication and contact the prescriber 3. have the pt. go to the hospital ER immediately 4. alert the pt. to the possibility of seizures and appropriate precautions

4. alert the pt. to the possibility of seizures and appropriate precautions

CH 13 The nurse is providing medication education to a patient who has been prescribed lithium to stabilize mood. Which early signs and symptoms of toxicity should the nurse stress to the pt? SELECT ALL THAT APPLY. 1. increased attentiveness 2. getting up at night to urinate 3. improved vision 4. an upset stomach for no apparent reason 5. Shaky hands that make holding a cup difficult

4. an upset stomach for no apparent reason 5. Shaky hands that make holding a cup difficult

CH 13 Luc's family comes home one evening to find him extremely agitated and they suspect in a full manic episode. the family calls emergency medical services. while one medic is talking with luc and his family, the other medic is counting something on his desk. what is the medic most likely counting? 1. Hypodermic needles 2. Fast food wrapper 3. empty soda cans 4. energy drink containers

4. energy drink containers

A nurse taught a client about important precautions associated with a new prescription. Afterward, the client accurately summarized major self-management strategies associated with this drug. Which step of the nursing process applies to the client's summarization? 1. Assessment 2. Analysis 3. Planning/outcomes identification 4. Intervention 5. Evaluation

5. Evaluation - Evaluation of the individual's response to treatment should be systematic, ongoing, and criteria based. Supporting data are included to clarify the evaluation.

Tort law

A civil wrong (as opposed to a criminal wrong) for which money damages may be collected by the injured party (plaintiff) from the responsible party (the defendant)

CH 12. Currently what is understood to be the causation of schizophrenia? A. A combination of inherited and nongenetic factors B. Deficient amounts of the neurotransmitter dopamine C. Excessive amounts of the neurotransmitter serotonin D. Stress related and ineffective stress management skills

A combination of inherited and nongenetic factors - Causation is a complicated matter. Schizophrenia most likely occurs as a result of a combination of inherited genetic factors and extreme nongenetic factors (e.g., viral infection, birth injuries, nutritional factors) that can affect the genes governing the brain or directly injure the brain

What is the "Recovery Model?"

A concept of healing and transformation enabling a person with mental illness to live a meaningful life in the community while striving to achieve his or her full potential

writ of habeas corpus

A court order that requires police to bring a prisoner to court to explain why they are holding the person

Dissociative Fugue Disorder

A disorder marked by episodes of amnesia in which a person is unable to recall some or all of his or her past and is confused about his or her identity; a new identity may be formed in which the person suddenly and unexpectedly travels away from home.

Central Serotonin Syndrome

A rare and life threatening event associated with the SSRIs, thought to be related to over activation of the central serotonin receptors, caused by either too high a dose or interactions with other drugs.

disenfranchised grief

A situation in which certain people, although they are bereaved, are prevented from mourning publicly by cultural customs or social restrictions. Person cannot grieve publicly; miscarriage, suicide; no structure or format to publicly grieve

CH 32. The term dual diagnosis refers to having a severe mental illness and what other dysfunctional behavior? A substance abuse problem Medication noncompliance A personality disorder HIV infection

A substance abuse problem - Dual diagnosis is the term used to identify a client with severe mental illness and a substance abuse problem. Both problems must be treated if the client is to be successfully rehabilitated. None of the other options reflect an accurate description of the term dual diagnosis

Cognitive theory

A theory of human development that focuses on changes in how people think over time. According to this theory, our thoughts shape our attitudes, beliefs, and behaviors.

CH 27. Which assessment finding is the best predictor of violence in a newly admitted client? A. A recent assault on a drinking companion B. A family history of bipolar disorder C. The nurse's subjective feeling that the client is uncooperative D. A childhood history of being bullied at school

A. A recent assault on a drinking companion - The best predictor of violence is past episodes of violent behavior. None of the remaining options have the predictability of a previous demonstration of anger

CH 26. What is the typical response when a stressful event occurs and the individual is unable to resolve the situation by using their usual coping strategies? A. A state of emotional disorder results in trial-and-error problem solving. B. They will withdraw and act as though the problem does not exist. C. They develop severe personality disorganization. D. Their distress often causes them to resort to planning suicide.

A. A state of emotional disorder results in trial-and-error problem solving. - The description is that of the second stage of crisis, according to accepted crisis theory. In this stage the individual will adopt the trial-and-error method of problem solving since their usual strategies are no longer effective

CH 16. Which statement about structural dissociation of the personality is true? A. An organic basis exists for this type of disorder. B. Nurses perceive clients with this disorder as easy to care for. C. No known link exists between this disorder and early childhood loss or trauma. D. This disorder results in a split in the personality causing a lack of integration.

A. An organic basis exists for this type of disorder. - The theory of structural dissociation of the personality proposes that patients with complex trauma have different parts of their personality, the apparently normal part and the emotional part, that are not fully integrated with each other. Each part has its own responses, feelings, thoughts, perceptions, physical sensations, and behaviors. These different parts may not be aware of each other, with only one dominant personality operating depending on the situation and circumstance of the moment. None of the other options are accurate statements regarding this disorder

CH 10. which nursing assessments are directed at monitoring a pts. flight or fight response? select all that apply. A. Blood pressure B. heart rate C. respiratory rate D. abdominal pain E. dilated pupils

A. Blood pressure B. heart rate C. respiratory rate E. dilated pupils

CH 16. What information should the nurse give to the family of a client who has had a dissociative episode? A. Dissociation is a method for coping with severe stress. B. Dissociation suggests the possibility of early dementia. C. Brief periods of psychotic behavior may occur. D. Ways to intervene to prevent self-mutilation and suicide attempts.

A. Dissociation is a method for coping with severe stress. - Childhood physical, sexual, or emotional abuse and other traumatic events are associated with adults experiencing dissociative symptoms. None of the other options are true.

CH 15. This theorist believed anxiety is linked to emotional distress caused when early needs go unmet or disapproval is experienced (interpersonal theory). Also suggested anxiety is contagious, transferring from mother to infant. A. Harry Stack Sullivan B. Walter Cannon (1871-1945) C. Hans Selye (1907-1982) D. Sigmund Freud

A. Harry Stack Sullivan

A nurse spends extra time with a client who has personality features similar to the nurse's estranged spouse. Which aspect of countertransference is most likely to result? A. Overinvolvement B. Misuse of honesty C. Indifference D. Rescue

A. Overinvolvement

CH 26. Which assumption serves as a foundation for the use of crisis intervention? A. The individual is mentally healthy but in a state of disequilibrium. B. Long-term dysfunctional adjustment can be addressed by crisis intervention. C. An anxious person is unlikely to be willing to try new problem-solving strategies. D. Crisis intervention nurses need to remain passive as the client deals with the crisis.

A. The individual is mentally healthy but in a state of disequilibrium. - The patient in a crisis situation is assumed to be mentally healthy, to have functioned well in the past, and to be presently in a state of disequilibrium. None of the other options present correction foundational statements for crisis intervention

CH 12. to provide effective care for the pt. diagnosed with schizophrenia, the nurse should frequently assess for which associated condition? select all that apply A. alcohol use disorder B. major depressive disorder C. stomach cancer D. polydipsia E. metabolic syndrome

A. alcohol use disorder B. major depressive disorder D. polydipsia E. metabolic syndrome

CH 12. Which characteristic in an adolescent female is sometimes associated with the prodromal phase of schizophrenia? A. always afraid another student will steal her belongings B. an unusual interest in numbers and specific topics C. demonstrates no interest in athletics or organized sports D. appears more comfortable among males

A. always afraid another student will steal her belongings

CH 15. which pt. has an increased risk for the development of anxiety and will require frequent assessment by the nurse? select all that apply A. exacerbation of asthma signs and symptoms B. history of peanut and strawberry allergies C. history of chronic obstructive pulmonary disease D. current treatment for unstable angina pectoris E. history of traumatic brain injury

A. exacerbation of asthma signs and symptoms C. history of chronic obstructive pulmonary disease D. current treatment for unstable angina pectoris E. history of traumatic brain injury

CH 12. kyle, a pt. with schizophrenia, began to take the first-generation antipsychotic haloperidol (Haldol) last week. one day you find him sitting very stiffly and not moving. he is diaphoretic, and when you ask if he is okay he seems unable to respond verbally. his vital signs are: BP 170/100, P 110, T 104.2. what is the priority nursing intervention? select all that apply. A. hold his medication and contact his prescriber B. wipe him with a washcloth wet with cold water or alcohol C. administer a medication such as benztropine IM to correct this dystonic reaction D. reassure him that although there is no treatment for his tardive dyskinesia, it will pass. E. hold his medication for now and consult his prescriber when he comes to the unit later today.

A. hold his medication and contact his prescriber B. wipe him with a washcloth wet with cold water or alcohol

CH 10. first responders and emergency department healthcare providers often use dark humor in an effort to: A. reduce stress and anxiety B. relive the experience C. rectify moral distress D. alert others to stress

A. reduce stress and anxiety

CH 12. which nursing intervention is particularly well chosen for addressing a population at high risk for developing schizophrenia? A. screening a group of males between the ages of 15 and 25 for early symptoms B. forming a support group for females aged 25 to 35 who are diagnosed with substance use issues C. providing a group for patients between the ages of 45 and 55 with information on coping skills that have proven to be effective D. educating the parents of a group of developmentally delayed 5 to 6 year olds on the importance of early intervention

A. screening a group of males between the ages of 15 and 25 for early symptoms

Cyclothymia is commonly misdiagnosed for what?

ADHD

Central Serotonin Syndrome manifestations

Abdominal pain diarrhea delirium fever tachycardia elevated BP irritability hostility mood change

Cultural Skill

Ability to perform a cultural assessment in a sensitive way, Use professional medical interpreter to ensure meaningful communication, Use culturally sensitive assessment tools. Goal, A mutually agreeable therapeutic plan .Culturally acceptable, Capable of producing positive outcomes

Characteristics of Conduct disorder?

Abnormal aggression Coercion One of the most frequently diagnosed disorders of childhood and adolescence

outcome identifications for abuse?

Abuse protection Abuse recovery Abuse cessation

what does recovery focus on?

Achieving goals of patient's choosing Leading increasingly productive and meaningful lives

define aggression

Action or behavior that results in verbal or physical attack. Not always inappropriate and can be necessary for self-protection.

4 cardinal features of Delirium?

Acute Onset and Fluctuating Course - levels of orientation might change over the course of the delirium Inattention - easily distracted by irrelevant stimuli Disorganized Thinking - confusion Disturbance of Consciousness - hallucinations, delusions, illusions

3 Phases of implementation for major depression

Acute phase (6 to 12 weeks) Continuation phase (4 to 9 months) Maintenance phase (1 year or more)

Signs of advanced lithium toxicity and range?

Advanced (around 1.5-2.0 mEq/L) Coarse hand tremor Persistent gastrointestinal upset Mental confusion Muscle hyperirritability Incoordination

Agranulocytosis (schizophrenia)

Agranulocytosis - signs and symptoms of infection, more susceptible to infection. Watch for fever

other crisis theories (Aguilero & Mesnick and Roberts)

Aguilero & Mesnick (1970's) - nursing framework Roberts (2005) - 7 stage model

What to do if pt is detoxing from alcohol and vitals begin to rise?

Alcohol is a CNS depressant so if the patient is detoxing and vitals begin rising - give patient another CNS depressant to keep them in safe zone until alcohol levels come down/regulate and vitals stabilize

how long do Antipsychotic medications for schizophrenia take to take effect?

All usually take 2 - 6 weeks for full effect All increase mortality in elderly adults with dementia

Healthy defense mechanisms

Altruism Sublimation Humor Suppression compensation introjection

HEALTHY defense mechanisms

Altruism Sublimation Suppression Humor

Is delirium usually PRIMARY or SECONDARY?

Always secondary to another physiological condition - need to figure out underlying cause (differential diagnosis)

Red flag for an eating disorder?

Amenorrhea - especially ANOREXIA

Cardinal symptoms of Alzheimer's Disease?

Amnesia or memory impairment Aphasia: loss of language ability Apraxia: loss of purposeful movement in the absence of motor or sensory impairment Agnosia: loss of sensory ability to recognize objects Disturbances in executive functioning - cant plan or organize - no abstract thinking

CH 29. How is the act of rape is best described? An act of violence using sex as the weapon Assault by a stranger on an unsuspecting victim Sexual desire satisfied inappropriately An act prompted by early childhood neglect

An act of violence using sex as the weapon - Rape is a violent crime. Sex is only the medium for perpetrating the crime. None of the remaining options accurately and thoroughly describe the act of rape

Panic Disorder

An anxiety disorder marked by unpredictable minutes-long episodes of intense dread in which a person experiences terror and accompanying chest pain, choking, or other frightening sensations.

define anger

An emotional response to frustration of desires, a threat to one's needs (emotional or physical), or a challenge

Therapeutic Milieu

An environment that provides client the opportunity to interact with staff and other clients. It gives the client the opportunity to practice interpersonal skills, provide feedback to peers about behavior, and work together to develop problem-solving skills.

CH 12. Which of the following would be assessed as a negative symptom of schizophrenia? A. Anhedonia B. Hostility C. Agitation D. Hallucinations

Anhedonia - Negative symptoms refer to deficits that characterize schizophrenia. They include the crippling symptoms of affective blunting (lack of facial expression), anergia (lack of energy), anhedonia (inability to experience happiness), avolition (lack of motivation), poverty of content of speech, poverty of speech, and thought blocking

Pharmacological interventions for Bulimia

Antianxiety/Antidepressant/Antipsychotic (Geodon) - Can help with OCD thoughts/behaviors - SSRIs (fluoxetine at higher dose than normally prescribed) - Topamax for binge suppression

Adjuncts to Antipsychotic Drug Therapy (schizophrenia)

Antidepressants Mood stabilizing agents - help in a manic phase Benzodiazepines - sedation, reduce anxiety

outcomes for anxiety disorder treatment

Anxiety self control Coping Self esteem Knowledge: disease process

situational crisis situations

Arises from events that are: - Extraordinary - External rather than internal - Often unanticipated Examples: job loss, divorce, illness, break-up

CH 32. A client who has been prescribed an antipsychotic medication comes to the clinic 3 days after his scheduled visit and demonstrates evidence of restlessness and agitation. He states, "My medicine ran out, and I didn't remember where to get more." The client's case manager should initially implement which intervention to support medication adherence? Arrange to have the client's nursing care plan reflect the need for a medication change Arrange for the client to see his psychiatrist as soon as the psychiatrist has an open appointment. Arrange for a dose of the client's medication immediately. Arrange for the client to get to the nearest emergency department for treatment.

Arrange for a dose of the client's medication immediately. - The role of the case manager is to coordinate access to psychiatric treatment, housing, rehabilitation or work setting, socialization, and medical care. The client's immediate need for medication is best addressed by arranging for an immediate dose. None of the other options addresses the client's needs as effectively

Evidence-Based Treatment Approaches for SMI?

Assertive community treatment (ACT) Cognitive-behavioral therapy (CBT) Cognitive-enhancement therapy (CET) Family support and partnership Social skills training Supportive psychotherapy Vocational rehabilitation and related

Ethnocentrism

Assuming one's own beliefs, values, and practices are the best, preferred, or only way

Anticholinergic toxicity (schizophrenia) symptoms and treatment

At greater risk if on multiple meds at the same time Worsening out of the blue symptoms: - Non reactive pupils - Hot, dry, red skin - Hyperpyrexia without diaphoresis - Tachycardia - Agitation - Delirium - Urinary retention treatment: - Contact doctor - Hold meds - Emergency cooling measures - Urinary cath prn - Sedation prn

Bipolar I

At least one episode of mania alternating with major depression

MAOIs are indicated for what time of pts?

Atypical or drug resistant depressions

foods to stay away from that contain tyramine?

Avocados fermented soybean fermented / smoked meats dried or cured fish / fermented / smoked ALL CHEESES figs / bananas

DSM-5 AXIS 1

Axis I: Mental disorder that is the focus of treatment

DSM-5 AXIS II

Axis II: Personality disorders and mental retardation

DSM-5 AXIS III

Axis III: General medical disorder relevant to the mental disorder in axis I

DSM-5 AXIS IV

Axis IV: Psychosocial and environmental problems

DSM-5 AXIS V

Axis V: Global Assessment of Functioning (GAF)

CH 15. which statement(s) made by the nurse demonstrates an understanding of the effective use of relaxation therapy for anxiety management? select all that apply A. "relaxation therapys main goal is to prevent exhaustion by removing muscle tension" B. " muscle relaxation promotes the relaxation response" C. "show me how you learned to deep breathe in yesterdays therapy session" D. "youve said that going to groups makes you nervous so lets start relaxing now" E. "ive given you written descriptions of the various relaxation exercises for you to review"

B. " muscle relaxation promotes the relaxation response" C. "show me how you learned to deep breathe in yesterdays therapy session" D. "youve said that going to groups makes you nervous so lets start relaxing now" E. "ive given you written descriptions of the various relaxation exercises for you to review"

CH 26. Which statement would suggest to the crisis intervention nurse the need to arrange for hospitalization of a client? A. "I'm feeling overwhelmed by all that has happened, and I need help sorting it out." B. "I see no solution for this situation if nothing changes by tomorrow." C. "There are three possibilities that might help, but I can't decide what to do." D. "I feel a little calmer than yesterday at this time, but things are still very difficult."

B. "I see no solution for this situation if nothing changes by tomorrow." - Whenever the client presents a danger to himself or herself or others, hospitalization must be considered. Such a danger may exist if the client expresses hopelessness as in the correct option. None of the other options express hopelessness.

CH 10. which question is focused on the assessment of an individual's personal ability to manage stress? select all that apply. A. "have you ever been diagnosed with cancer?" B. "do you engage in any hobbies now that you have retired?" C. "have you been taking your antihypertensive medication as it is prescribed?" D. "who can you rely on if you need help after youre discharged from the hospital?" E. "what do you do to help manage the demands of parenting a 4 year old and a newborn?"

B. "do you engage in any hobbies now that you have retired?" D. "who can you rely on if you need help after youre discharged from the hospital?" E. "what do you do to help manage the demands of parenting a 4 year old and a newborn?"

CH 10. what assessment question is focused on identifying a long term consequence of chronic stress on physical health? A. "do you have any problems with sleeping well?" B. "how many infections have you had in the past 6 months?" C. "how much moderate exercise do you engage in on a weekly basis?" D. "what management techniques do you regularly use to manage your stress?"

B. "how many infections have you had in the past 6 months?"

CH 10. hugo has a fraternal twin named franco who is unaffected by mental illness even though they were raised in the same dysfunctional household. franco asks the nurse "why hugo and not me?" the nurse replies: A. "your father was probably less abusive to you" B. "hugo likely has a genetic vulnerability" C. "you probably ignored the situation" D. "hugo responded to perceived threats by focusing on an internal world"

B. "hugo likely has a genetic vulnerability"

CH 15. A client's daughter states, "My mother lives with me since my dad died 6 months ago. For the past couple of months, every time I need to leave the house for work or anything else, Mom becomes extremely anxious and cries that something terrible is going to happen to me. She seems OK except for these times, but it's affecting my ability to go to work." This information supports that the client may be experiencing which anxiety-related disorder? A. Panic disorder B. Adult separation anxiety disorder C. Agoraphobia D. Social anxiety disorder

B. Adult separation anxiety disorder - People with separation anxiety disorder exhibit developmentally inappropriate levels of concern over being away from a significant other. There may also be fear that something horrible will happen to the other person. Adult separation anxiety disorder may begin in childhood or adulthood. The scenario doesn't describe panic disorder. Agoraphobia is characterized by intense, excessive anxiety or fear about being in places or situations from which escape might be difficult or embarrassing or in which help might not be available. Social anxiety disorder, also called social phobia, is characterized by severe anxiety or fear provoked by exposure to a social or a performance situation that will be evaluated negatively by others

A client tells the nurse, "I have something secret to tell you, but you can't tell anyone else." The nurse agrees. What is the likely consequence of the nurse's action? A. Healthy feelings of sympathy by the nurse toward the client. B. Blurred boundaries in the nurse-client relationship. C. Improved rapport between the nurse and client. D. Enhanced trust between the nurse and client.

B. Blurred boundaries in the nurse-client relationship.

CH 15. which medication should the nurse be prepared to educate pts. on when they are prescribed a selective serotonin reuptake inhibitor (SSRI) for panic attacks? A. Alprazolam (Xanax) B. Fluoxetine (Prozac) C. Clonazepam (Klonopin) D. Venlafaxine (Effexor)

B. Fluoxetine (Prozac)

CH 26. Considering client function, what is the expected outcome at the conclusion of crisis intervention therapy? A. Function is higher level than before the crisis. B. Function is at the precrisis level. C. Function is only marginally below the precrisis level. D. Function is occurring without aid from identified support systems.

B. Function is at the precrisis level. - The intent of crisis intervention is to return the individual to the precrisis level of functioning. While this goal is not always attainable, it remains the expected outcome of crisis intervention therapy

CH 12. Nico, a 22-year-old patient, is diagnosed with schizophrenia. Which of the following symptoms would alert a provider to a possible diagnosis of schizophrenia in a 22-year-old male client? A. Excessive sleeping with disturbing dreams B. Hearing voices telling him to hurt his roommate C. Withdrawal from college because of failing grades D. Chaotic and dysfunctional relationships with his family and peers

B. Hearing voices telling him to hurt his roommate - People diagnosed with schizophrenia all have at least one psychotic symptom such as hallucinations, delusional thinking, or disorganized speech. The other options do not describe schizophrenia but could be caused by a number of problems.DIF: Cognitive Level: Apply (Application)REF: page 17TOP: Nursing Process

CH 16. Which behavior best supports the diagnosis of posttraumatic stress disorder (PTSD) in a 4-year-old child? A. Overeating B. Hypervigilance C. A drive to be perfect D. Passivity

B. Hypervigilance - PTSD in preschool children may manifest as irritability, aggressive or self-destructive behavior, sleep disturbances, problems concentrating, and hypervigilance. None of the other options are characteristic of PTSD in a young child

CH 27. Which neurotransmitter imbalance has been shown to be related to impulsive aggression? A. Low levels of ã-aminobutyric acid B. Low levels of serotonin C. High levels of norepinephrine D. High levels of acetylcholine

B. Low levels of serotonin - Low serotonin levels have been implicated in several research studies as being a factor in impulsive aggression. Research does not support any of the other options

CH 27. Which nursing diagnosis is the priority when planning care for a client who displays considerable anger and occasional aggression? A. Social isolation B. Risk for other-directed violence C. Ineffective coping: overwhelmed D. Ineffective coping: maladaptive

B. Risk for other-directed violence - Risk for other-directed violence is the priority diagnosis. The nurse then must determine which of two other diagnoses—ineffective coping: overwhelmed or ineffective coping: maladaptive—is appropriate. Social isolation is not an initial concern

CH 26. What type of crisis is a newly unemployed person most likely to experience? A. Reactive B. Situational C. Maturational D. Unexpected

B. Situational - Situational crises arise from external sources. Examples are death of a loved one, divorce, marriage, or a change in health status. Unemployment is not generally associated with any of the other options

CH 12. Which nursing intervention is designed to help a schizophrenic client minimize the occurrence of a relapse? A. Schedule the client to attend group therapy that includes those who have relapsed. B. Teach the client and family about behaviors associated with relapse. C. Remind the client of the need to return for periodic blood draws to minimize the risk for Relapse. D. Help the client and family adapt to the stigma of chronic mental illness and periodic relapses.

B. Teach the client and family about behaviors associated with relapse. - By knowing what behaviors signal impending relapse, interventions can be quickly invoked when the behaviors occur. The earlier the intervention, the greater the likelihood that a recurrence can be averted. None of the other options are effective interventions when considering relapse prevention

CH 15. A 72-year-old patient diagnosed with Parkinson's disease is demonstrating behaviors associated with anxiety and has had several falls lately and is reluctant to take medications as prescribed. When his provider orders lorazepam, 1 mg PO bid, the nurse questions the prescription based primarily on what fact? A. The client may become addicted faster than younger patients. B. The client is at risk for falls. C. The client has a history of nonadherence with medications. D. The client should be treated with cognitive therapies because of his advanced age.

B. The client is at risk for falls. - An important nursing intervention is to monitor for side effects of the benzodiazepines, including sedation, ataxia, and decreased cognitive function. In a patient who has a history of falls, lorazepam would be contraindicated because it may cause sedation and ataxia leading to more falls. There is no evidence to suggest that elderly patients become addicted faster than younger patients. A history of nonadherence would not lead to you to question this drug order. Medication and other therapies are used congruently with all age levels

CH 10. what is a Short-Acting Sedative-Hypnotic Sleep Agents ("Z-hypnotics") for anxiety A. Diazepam (Valium) B. Zolpidem (Ambien) C.Alprazolam (Xanax) D.Lorazepam (Ativan)

B. Zolpidem (Ambien)

CH 10. hugo is 21 and diagnosed with schizophrenia. his history includes significant turmoil as a child and adolescent. hugo reports his father was abusive and routinely beat him, all of his siblings, and his mother. hugos early exposure to stress most likely: A. made him resilient to stressful situations B. increased his future vulnerability to psychiatric disorders C. developed strong survival skills D. shaped his nurturing nature

B. increased his future vulnerability to psychiatric disorders

CH 12. The purpose of the Abnormal Involuntary Movement Scale (AIMS) assessment on a persistently mentally ill client who has been diagnosed with schizophrenia is early detection of A. acute dystonia. B. tardive dyskinesia. C. cholestatic jaundice. D. pseudoparkinsonism.

B. tardive dyskinesia. - An AIMS assessment should be performed periodically on clients who are being treated with antipsychotic medication known to cause tardive dyskinesia. This tool is not used to assess or monitor any of the other options

who discovered cognitive behavioral therapy?

BECK

Cognitive Behavioral Therapy

BECK Test distorted beliefs and change thinking patterns then behavior changes.

Behavioral therapy for child and adolescents with impulse control disorders is based on...?

Based on principle that behavior that is rewarded is more likely to be repeated

Nursing considerations for pt with Histrionic PD?

Be aware that they may use seductive behavior as response to stress Keep relationship professional

Assessment tools for major depression?

Beck depression inventory geriatric depression scale

Treatment for DT?

Benzodiazepines

other classes of Pharmacological interventions for anxiety

Beta-blockers, antihistamines, anticonvulsants, and antipsychotics

milieu therapy

Bettelheim (1948) Use of total environment People, setting, structure, and emotional climate all important to healing

Nursing considerations for pt with borderline PD?

Beware of manipulative behaviors Have clear/consistent boundaries Remain neutral

Biggest complaints from pts when taking Lithium?

Biggest complaint is lack of energy, creativity, motivation —> so patient stops taking meds

Other characteristics of Bulimia?

Binge eating Purging or non-purging type Low self-esteem and self-worth Self-induced vomiting or laxative/diuretic use Impulsive behaviors Increased anxiety Chemical dependency is common

Childhood conduct disorder in males vs. females

Boys: fight, steal, vandalize Girls: truant, run away, abuse substances, engage in prostitution

Area of brain of ALZHEIMER pt that develops most plaques and tangles?

Brain regions important in memory & thinking and planning develop more plaques and tangles.

CH 12. A client diagnosed with paranoid schizophrenia refuses food, stating the voices are saying the food is contaminated and deadly. Which response should the nurse provide to this client statement? A. You are safe here in the hospital; nothing bad will happen to you." B. "The voices are wrong about the hospital food. It is not contaminated." C. "I understand that the voices are very real to you, but I do not hear them." D. "Other people are eating the food, and nothing is happening to them.

C. "I understand that the voices are very real to you, but I do not hear them." - This reply acknowledges the client's reality but offers the nurse's perception that he or she is not experiencing the same thing. This is the only option that provides such support.REF: 208

CH 15. the nurse is providing care for a patient demonstrating behaviors associated with moderate levels of anxiety. what question should the nurse ask initially when attempting to help the pt. deescalate their anxiety? A. "do you know what will help you manage your anxiety? B. "do you need help to manage your anxiety?" C. "can you identify what was happening when your anxiety began to increase?" D. "are you feeling anxious right now?"

C. "can you identify what was happening when your anxiety began to increase?"

CH 12. which therapeutic communication statement might a psychiatric-mental health nurse when a pt's nursing diagnosis is altered thought processes? A. "i know you say you can hear voices, but i cannot hear them." B. "stop listening to the voices, they are NOT real." C. "you say you hear voices, what are they telling you?" D. "please tell the voices to leave you alone for now."

C. "you say you hear voices, what are they telling you?"

CH 26. A crisis is so acutely uncomfortable to the individual that it is likely to self-resolve in what time frame? A. 1 to 10 days. B. 1 to 3 weeks. C. 4 to 6 weeks. D. 3 to 4 months.

C. 4 to 6 weeks. - At 4 to 6 weeks, the individual is making accommodations and adjustments to relieve anxiety, and the crisis is no longer a crisis.

CH 27. Which event is most likely to contribute to a client's escalating anger? A. Watching violence on television B. Another client's depressed mood C. A staff member challenging them D. A staff member asking them to help another client

C. A staff member challenging them - Punitive, threatening, accusatory, or challenging statements to the client should be avoided since they are likely to escalate the client's anger. None of the other options is as likely to escalate existing angry behavior

CH 16. which disorder is precipitated by a stressful event that is not considered traumatic? A. schizophrenia B. PTSD C. Adjustment disorder D. dissociate disorder

C. Adjustment disorder

CH 12. A client, who has been prescribed clozapine 6 weeks ago, reports flulike symptoms including a fever and a very sore throat, the nurse should initiate which nursing intervention? A. Suggest that the client take something for the fever and get extra rest. B. Advise the physician that the client should be admitted to the hospital. C. Arrange for the client to have blood drawn for a white blood cell count. D. Consider recommending a change of antipsychotic medication.

C. Arrange for the client to have blood drawn for a white blood cell count. - Antipsychotic medications may cause agranulocytosis, the first manifestation of which may be a sore throat and flulike symptoms

CH 16. A child who was physically and sexually abused is at great risk for demonstrating which characteristic? A. Depression B. Suicide attempts C. Bullying and abusing others D. Becoming active in a gang

C. Bullying and abusing others - Children who have been abused are at risk for abusing others, as well as for developing dysfunctional patterns in close interpersonal relationships. While the other characteristics may occur, none are as characteristic as the correct option

CH 15. The nurse is caring for a patient on day 1 post surgical procedure. The patient becomes visibly anxious and short of breath, and states, "I feel so anxious! Something is wrong!" What action should the nurse take initially in response to the patient's actions? A. Reassure the patient that what they are feeling is normal anxiety and do deep breathing exercises with her. B. Use the call light to inquire whether the patient has been prescribed prn anxiety medication. C. Call for staff help and assess the client's vital signs. D. Reassure the patient that you will stay until the anxiety subsides.

C. Call for staff help and assess the client's vital signs. - In anxiety caused by a medical condition, the individual's symptoms of anxiety are a direct physiological result of a medical condition, such as hyperthyroidism, pulmonary embolism, or cardiac dysrhythmias. In this case, Lana is postoperative and could be experiencing a pulmonary embolism, as evidenced by the shortness of breath and anxiety. She needs immediate evaluation for any serious medical condition. The other options would all be appropriate after it has been determined that no serious medical condition is causing the anxiety

CH 12. A client diagnosed with disorganized schizophrenia would have greatest difficulty with the nursing intervention? A. Interacting with a neutral attitude B. Using concrete language C. Giving multistep directions D. Providing nutritional supplements

C. Giving multistep directions - The thought processes of the client with disorganized schizophrenia are severely disordered, and severe perceptual problems are present, making it extremely difficult for the client to understand what others are saying. All communication should be simple and concrete and may need to be repeated several times. Ineffective organizational skills would not be a primary factor considering the other options

CH 26. Which nursing action directed toward a client in crisis demonstrates signs of a problematic nurse-client relationship? A. Offering to change the time of the counseling session for the second time in 3 weeks B. Experiencing frustration about the decisions the client is making C. Giving the client a personal number to call when they "need to talk" D. Suggesting that the client attend an extra counseling session each month

C. Giving the client a personal number to call when they "need to talk" - Giving a client one's personal telephone number is a reaction to the nurse's need to be needed and undermines the client's sense of self-reliance. None of the other behaviors demonstrate a breakdown in the nurse-client behavior but rather reactions to commonly experienced issues within that relationship

Ch. 10 which theorist discovered the General Adaptation Syndrome (GAS) which expanded theory of flight-or-flight? A. Walter Cannon (1871-1945 B. Sigmund Freud C. Hans Selye (1907-1982) D. Harry Stack Sullivan

C. Hans Selye (1907-1982)

CH 12. The nurse is planning long-term goals for a 17-year-old male client recently diagnosed with schizophrenia. Which statement should serve as the basis for the goal-setting process? A. If treated quickly following diagnosis, schizophrenia can be cured. B. Schizophrenia can be managed by receiving treatment only at the time of acute exacerbations. C. Patients with schizophrenia often do not fully respond to treatment and have residual symptoms and varying degrees of disability. D. If patients with schizophrenia stay on their drug regimen, they usually lead fully productive lives with no further symptoms.

C. Patients with schizophrenia often do not fully respond to treatment and have residual symptoms and varying degrees of disability. - Unfortunately, in most cases, schizophrenia does not respond fully to available treatments; it leaves residual symptoms and causes varying degrees of dysfunction or disability. The other options are all untrue of schizophrenia.

CH 12. A teenaged client is being discharged from the psychiatric unit with a prescription for risperidone. The nurse providing medication teaching to the client's mother should provide which response when asked about the risk her son faces for extrapyramidal side effects (EPSs)? A. All antipsychotic medications have an equal chance of producing EPSs. B. Newer antipsychotic medications have a higher risk for EPSs. C. Risperidone is a newer antipsychotic medication and has a lower risk of EPSs than older antipsychotics. D. Advise the mother to ask the provider to change the medication to clozapine instead of risperidone.

C. Risperidone is a newer antipsychotic medication and has a lower risk of EPSs than older antipsychotics. - Risperidone is a newer, atypical antipsychotic. All newer antipsychotic medications have a lower incidence of EPSs than older, traditional antipsychotics. The other responses are untrue. There is no reason to advise a medication change at this time.

CH 16. A client diagnosed with post-traumatic stress disorder (PTSD) shows little symptom improvement after being prescribed a selective serotonin reuptake inhibitor (SSRI). The nurse expects that which medication will be prescribed next? A. Beta blocker B. Barbiturate C. Tricyclic antidepressant (TCA) D. Sedative

C. Tricyclic antidepressant (TCA) - TCAs or mirtazapine (Remeron) may be prescribed if SSRIs or SNRIs are not tolerated or do not work. None of the other options would be the next consideration

CH 12. tomas is a 21-year old male with a recent diagnosis of schizophrenia. tomas's nurse recognizes that self-medicating with excessive alcohol is common in this disease and can co-occur along with: A. generally good health despite the mental illness B. an aversion to drinking fluids C. anxiety and depression D. the ability to express his needs

C. anxiety and depression

CH 10. the pt. you are assigned unexpectedly suffers a cardiac arrest. during this emergency situation, your body will produce a large amount of: A. carbon dioxide B. growth hormone C. epinephrine D. aldosterone

C. epinephrine

CH 10. your 39 year old patient, samantha, who was admitted with anxiety, asks you what the stress-relieving technique of mindfulness is. the best response is: A. mindfulness is focusing on an object and repeating a word or phrase while deep breathing B. mindfulness is progressively tensing, then relaxing, body muscles C. focusing on the here and now, not the past or future, and paying attention to what is going on around you D. a memory system to assist you in short term memory recall

C. focusing on the here and now, not the past or future, and paying attention to what is going on around you

CH 10. when considering stress, what is the primary goal of making daily entries into a personal journal? A. providing a distraction from the daily stress B. expressing emotions to manage stress C. identifying stress triggers D. focusing on one's stress

C. identifying stress triggers

CH 10. which behavioral therapy technique used gradual exposure? A. Modeling B. Flooding C. systematic desensitization D. thought stopping

C. systematic desensitization -pt. gradually introduced to a feared object or experience thru a series of steps, from least frightening to most (graduated exposure). Pt is taught to use a relaxing technique at each step when anxiety becomes overwhelming

Patient comes into ER with major depression and flu-like symptoms?

CENTRAL SEROTONIN SYNDROME

Advantage of MAOIs?

Can be used as a LAST RESORT MED

how long does it take for TCAs to take effect?

Can take up to 28 days for mood elevation (**6-8 weeks for full effect**)

Panic

Cannot focus at all Cannot problem solve 'may hallucinate or have delusions' , may feel numb, Cannot learn, irrational, may feel terror Exhaustion, behavior erratic, impulsive, Sleeplessness, severe shakes

who developed 4 phases of crisis?

Caplan (1964)

Potentially Beneficial Services or Treatment Approaches for SMI?

Case management Medication monitoring Assertive community treatment (ACT) Day programs Crisis intervention services Advance directives Consumer-run programs Wellness and recovery action plans (WRAP) Technology Exercise

additional characteristics of Cluster B personality disorders?

Challenges in self-direction Problems with empathy Overreactive and impulsive Danger (especially with cluster B) - Might self-harm or harm another or threaten Borderline - SPLITTING: all good or all bad; very good at manipulating to get what they want

Define "Dysthymia"

Chronic depressive syndrome Present for most of the day More days than not At least 2 years

what drug is said to induce agranulocytosis?

Clozaril - 2nd gen medication

Importance of "Cognition"

Cognition distinguishes living from existing gives meaning and importance connections between events/actions personality and sense of self

DELIRIUM is common in...?

Common in hospitalized patients, especially older adults - usually kicks in the second or third day in hospital

Emergency Care and Crisis Stabilization

Comprehensive emergency service model Hospital-based consultant model Mobile crisis team model Crisis stabilization/observation units

Culturally relevant care

Concept of mental health and illness is formed within a culture Deviance from cultural expectations can be defined as illness by other members of the group Diversity of United States and the rest of the world increasing Culturally relevant nursing becoming more important

Alzheimer's Assessment

Confabulation: Unconscious attempt to maintain self-esteem Perseveration: Repetition of phrase or behaviors

Ethical dilemma

Conflict between two or more courses of action, each with favorable and unfavorable consequences

Suppression defense mechanism

Consciously and intentionally pushing unpleasant feelings out of one's mind

CH 32. The role of a case manager working with severely and persistently mentally ill clients who are homeless would include which intervention? Administer medication Coordinate needed services Ensure that the clients are not rehospitalized Teach the clients to function independently

Coordinate needed services - Community mental health services are designed to provide outreach and case management for severely mentally ill persons who are homeless. A team approach is used to gain access to clients and connect them with the various services available to meet their needs. The role of the outreach worker is to be an advocate in all areas of client need and to foster client self-care. The role of the case manager does not include any of the other options

Function of the brainstem

Core - Regulates internal organs and vital functions Hypothalamus - Basic drives and link between thought and emotion and function of internal organs Brainstem - Processing center for sensory information

CH 12. A client diagnosed with schizophrenia states to the nurse, "My, oh my. My mother is brother. Anytime now it can happen to my mother." Your best response would be: A. "You are having problems with your speech. You need to try harder to be clear." B. "You are confused. I will take you to your room to rest a while." C. "I will get you a prn medication for agitation." D. "I'm sorry, I didn't understand that. Do you want to talk more about your mother as we did yesterday?"

D. "I'm sorry, I didn't understand that. Do you want to talk more about your mother as we did yesterday?" - The guidelines that are useful in communicating with a patient with disorganized or bizarre speech are to place the difficulty in understanding on yourself, not the patient, and look for themes that may be helpful in interpreting what the patient wants to say. Telling the patient he needs to try harder to be clearer is unrealistic since the patient would be unable do this. The other options are not useful in communicating with this patient and attempting to find common themes

CH 12. When a client diagnosed with paranoid schizophrenia tells the nurse, "I have to get away. The volmers are coming to execute me," an appropriate response for the nurse would be: A. "You are safe here. This is a locked unit, and no one can get in." B. "I do not believe I understand the word volmers. Tell me more about them." C. "Why do you think someone or something is going to harm you?" D. "It must be frightening to think something is going to harm you."

D. "It must be frightening to think something is going to harm you." - This response focuses on the client's feelings and neither directly supports the delusion nor denies the client's experience. Option A gives global reassurance. Option B encourages elaboration about the delusion. Option C asks for information that the client will likely be unable to answe

CH 15. How long does General Anxiety Disorder (GAD) typically last? A. 1-2 weeks B. 2-3 months C. 4-5 months D. 6 months or longer

D. 6 months or longer

CH 16. Which child is at greatest risk for developing attachment problems as a result of a neurobiological development? A. A 13-year-old male B. A 10-year-old female C. A 7-year-old male D. A 4-year-old female

D. A 4-year-old female - The developing brain is particularly vulnerable to adverse events because the most rapid brain development occurs in the first five years of life. The right hemisphere is involved in processing social-emotional information, promoting attachment functions, regulating body functions, and in supporting the individual in survival and in coping with stress. Since the right brain develops first and is involved with developing templates for relationships and regulation of emotion and bodily function, early attachment relationships are particularly important for healthy development and life-long health

CH 27. How is anger best defined? A. An unhealthy way of releasing anxiety B. Doing intentional harm to others C. An expression of conflict with others D. A normal response to a perceived threat

D. A normal response to a perceived threat - Anger is an emotional response to frustration of desires, a threat to one's needs (emotional or physical), or a challenge. It is a normal emotion that can even be positive when it is expressed in a healthy way. All the other options describe anger as being abnormal and/or dysfunctional.

CH 15. The nurse is providing teaching to a preoperative patient just before surgery. The patient is becoming more and more anxious and begins to report dizziness and heart pounding. The patient also appears confused and is trembling noticeably. Considering the scenario, what decision should the nurse make? A. To reinforce the preoperative teaching by restating it slowly. B. Have the patient read the teaching materials instead of providing verbal instruction. C. Have a family member read the preoperative materials to the patient. D. Do not attempt any further teaching at this time.

D. Do not attempt any further teaching at this time. - Patients experiencing severe anxiety, as the symptoms suggest, are unable to learn or solve problems. The other options would not be effective because you are still attempting to teach someone who has a severe level of anxiety.

CH 15. Which medication is FDA approved for treatment of anxiety in children? A. Sertraline B. Fluoxetine C. Clomipramine D. Duloxetine

D. Duloxetine - A few drugs are approved specifically for anxiety and obsessive-compulsive disorders in children and adolescents. The FDA approved the selective SNRI duloxetine (Cymbalta) in 2014 for children aged 7 to 17 years for generalized anxiety disorder. The FDA has approved four medications for use in children with obsessive-compulsive disorder. They are clomipramine (Anafranil), fluoxetine (Prozac), fluvoxamine (Luvox), and sertraline (Zoloft)

CH 26.What type of crises occur as an individual moves from one developmental level to another? A. Reactive B. Recurring C. Situational D. Maturational

D. Maturational - Maturational crises are normal states in growth and development in which specific new maturational tasks must be learned when old coping mechanisms are no longer effective. This challenge is not specifically associated with any of the other options.

CH 12. A client has reached the stable plateau phase of schizophrenia. What is the appropriate clinical planning focus for this client? A. Safety and crisis intervention B. Acute symptom stabilization C. Stress and vulnerability assessment D. Social, vocational, and self-care skills

D. Social, vocational, and self-care skills - During the stable plateau phase of schizophrenia, planning is geared toward client and family education and skills training that will help maintain the optimal functioning of schizophrenic individuals in the community. All the other options should have been handled previously.

CH 12. Tara and Aaron are twins who are both diagnosed with schizophrenia. Aaron was diagnosed at 23 years old and Tara at 31 years old. Based on your knowledge of early and late onset of schizophrenia, which of the following is true? A. Tara and Aaron have the same expectation of a poor long-term prognosis. B. Tara will experience more positive signs of schizophrenia such as hallucinations. C. Aaron will be more likely to hold a job and live a productive life. D. Tara has a better chance for positive outcomes because of later onset.

D. Tara has a better chance for positive outcomes because of later onset. - Female patients diagnosed with schizophrenia between the ages of 25 and 35 years have better outcomes than do their male counterparts diagnosed earlier. These two patients do not have the same expectation of a poor prognosis. There is no evidence suggesting that Tara will have more positive signs of schizophrenia. It is actually more unlikely that Aaron will be able to live a productive life because of his earlier onset, which has a poorer prognosis

CH 16. Parents express concern when their 5-year-old child, who is receiving treatment for cancer, keeps referring to an imaginary friend, Candy. Which response should the nurse provide to best address the parent's concerns? A. Children of this age usually have imaginary friends. B. It is nothing to worry about unless the child starts to socially isolate. C. The child needs more of their one-on-one attention. D. The imaginary friend is a coping mechanism the child is using.

D. The imaginary friend is a coping mechanism the child is using. - Often traumatized children feel responsible for what happened to them and are frightened by flashbacks, amnesia, or hallucinations that may be due to trauma. For example, a child may use imaginary friends as a coping mechanism. This option addresses the parents' concern most effectively

CH 27. Nurses coping with angry clients may find it helpful to remember that anger and aggression begin as feelings of which personal characteristic? A. Isolation B. Confidence C. Hopelessness D. Vulnerability

D. Vulnerability - The progression is vulnerability, perception of event as a threat, arousal, and then uneasiness and anxiety. Anger is not as influenced by any of the other options

CH 10. which antianxiety med does not cause dependence? A. Prozac B. Effexor C. cymbalta D. buSpar

D. buSpar

CH 15. to maximize the therapeutic effect, which lifestyle practice should the nurse discourage for a pt. who has recently been prescribed an antianxiety med? A. eating high protein foods B. using acetaminophen without first discussing it with a healthcare provider C. taking meds after eating dinner or while having a bedtime snack D. buying a large coffee with sugar and extra cream each morning on the way to work

D. buying a large coffee with sugar and extra cream each morning on the way to work

CH 12. which characteristic presents the greatest risk for injury to others by the pt. diagnosed with schizophrenia? A. depersonalization B. pressured speech C. negative symptoms D. paranoia

D. paranoia

CH 10. jackson has suffered from migraine headaches all his life. Fatima, his N.P., suspects muscle tension as a trigger for his headaches. Fatima teaches him a technique that promotes relaxation by using: A. biofeedback B. guided imagery C. deep breathing D. progressive muscle relaxation

D. progressive muscle relaxation

CH 12. a female pt. diagnosed with schizophrenia has been prescribed a first generation antipsychotic medication. what information should the nurse provide to the patient regarding her signs and symptoms? A. her memory problems will likely decrease B. depressive episodes should be less severe C. she will probably enjoy social interactions more D. she should experience a reduction in hallucinations

D. she should experience a reduction in hallucinations - 1st gen medications target the POSITIVE symptoms of schizophrenia; delusions and hallucinations

CH 12. when pt's diagnosed with schizophrenia suffer from anosognosia, they often refuse medication, believing that: A. medications provided are ineffective B. nurses are trying to control their minds C. the medications will make them sick D. they are not actually ill

D. they are not actually ill anosognosia - lack of realizing you have a problem

CH 12. gilbert, age 19, is described by his parents as a "moody child" with an onset of odd behavior about at age 14, which caused gilbert to suffer academically and socially. he has lost the ability to complete household chores, is reluctant to leave the house, and is obsessed with the locks on the windows and doors. due to gilbert's early and slow onset of what is now recognized as schizophrenia, his prognosis is considered: A. favorable with medication B. in the relapse stage C. improvable with psychosocial interventions D. to have a less positive outcome

D. to have a less positive outcome - factors associated with a less positive outcome include a slow onset (more than 2-3 years), younger age at onset, longer duration between first symptoms and first treatment, longer periods of untreated illness, and more negative symptoms.

assessment strategies for SMI?

Danger to self/others Hopelessness Signs of relapse Inadequate attention to self care Signs of treatment non-adherence Physical problems

When does delirium peak with DT and how long does it last?

Delirium peaks at 2 to 3 days after cessation of alcohol and lasts 2 to 3 days

As delirium worsens, what symptoms might you observe?

Delusions - false beliefs (someone is out to get them; not perceptual distortion) Illusions - stimulus is true but it is externally misrepresented (wavy pattern in the rug and they think it is snakes) make sure there is good lighting, eliminate shadows, cover mirrors Hallucinations - false internal misrepresentations; visual hallucinations common in delirium Perseveration - purposeless repetition of idea, words, emotions

Which coping mechanism is used excessively by clients diagnosed with bulimia nervosa to cope with their obsession with their body image? 1. Denial 2. Humor 3. Altruism 4. Projection

Denial ~ Denial of incongruence between body reality, body ideal, and body presentation is the mainstay of the client diagnosed with bulimia nervosa. None of the other mechanisms are as vital to their coping technique.

Stimulant (Crack / Cocaine) withdrawal symptoms

Depression paranoia lethargy anxiety insomnia nausea / vomiting sweating / chills

what are some psychological effects of sexual assault?

Depression Suicide Anxiety Fear Difficulties with daily functioning Low self-esteem Sexual dysfunction Somatic complaints

Therapeutic Strategies

Develop nurse-client relationship Use of psychosocial communication skills Counseling Advocate for the client Lead groups Health teaching Milieu therapy Psychobiological interventions

Seperation Anxiety Disorder (SAD)

Developmentally inappropriate levels of concern over being away from a significant other May also be a fear that something horrible will happen to the other person and will result in permanent separation Can also manifest in physical symptoms such as GI disturbances and headache

diathesis-stress model

Diathesis - biological predisposition Stress - environmental stress or trauma Most accepted explanation for mental illness Nature plus nurture Combination of genetic vulnerability and negative environmental stressors

antianxiety and hypnotic drugs: Benzodiazepines

Diazepam (Valium) Clonazepam (Klonopin) Alprazolam (Xanax) Lorazepam (Ativan)

CH 28. When interviewing an adult victim of abuse, what is the nurse's best approach when asking relevant questions? A. Confrontational and assertive B. Gentle but direct C. Direct and professional D. Sympathetic but detached

Direct and professional - Expressing strong emotion does not help the victim. A direct, honest, and professional manner of asking questions produces the best results. None of the other options are recognized as being effective in this type of interview situation.

Characteristics of Oppositional Defiant disorder?

Disorder usually evident before 8 years of age Stubborn Argumentative Testing of limits Refusal to accept blame Deviant behavior with authority figures No serious violations of basic rights of others Symptomatic behavior most evident at home

dissociative amnesia

Dissociative disorder characterized by the sudden and extensive inability to recall important personal information, usually of a traumatic or stressful nature.

Lazarus - distress vs. eustress

Distress - a negative draining energy that results in anxiety, depression, confusion, helplessness, hopelessness, and fatigue. Eustress - positive beneficial energy that motivates and results in feelings of happiness, hopefulness, and purposeful movement. It is the result of a positive perception toward a stressor.

Pharmacological Interventions for Alzheimer's Disease

Donepezil (Aricept) - most common (only taken once a day) ; early stages Memantine (Namenda) - for mild to moderate alzheimers Rivastigmine (Exelon) - can be given as a daily patch Galantamine (Razadyne) Tacrine (Cognex) - do not use as much anymore due to liver toxicity - START LOW GO SLOW

Characteristics of Histrionic PD?

Dramatic presentation of self - all life is a stage Excessive emotionality - to seek attention, love, admiration Overly concerned with impressing others Others experience this as smothering & destructive Behavior can be interpreted as coercive and attention seeking All suicide ideation must be taken seriously

Most common characteristics of cluster B PD?

Dramatic, emotional, erratic behavior Problems with impulse control

possible antipsychotic side effects?

Dry mouth Urinary retention and hesitancy Constipation Blurred vision Photosensitivity Dry eyes impotence in men Sedation Stiffness

who discovered rational emotive behavioral therapy?

ELLIS

rational emotive behavior therapy

ELLIS Eradicate irrational beliefs/recognize inaccurate thoughts. Focus on present - can't change past Help recognize thoughts are not accurate, sensible, or useful.

milieu therapy - patient is on suicide watch - how often do you check on them?

EVERY 15 MINUTES

Early signs of Lithium toxicity and range?

Early (around 1.5 mEq/L) Nausea Vomiting Diarrhea Thirst Polyuria Slurred speech Muscle weakness

Secondary prevention for abuse

Early intervention in abusive situations to minimize their disabling or long-term effects

ECT pre-procedure

Education about what it is Signed Informed Consent- why? Medical clearance (labs, EKG, spinal x-ray), remove dentures, NPO after midnight, no jewelry/nail polish so O2 can be monitored Meds Administered: IV anesthetic, muscle relaxant, med to decrease secretions Close monitoring of cardiac and respiratory function

Medical complications and effects of vomiting in bulimic pt?

Electrolyte levels Glucose level Thyroid function tests CBC ECG

Cognitive theorists?

Ellis Beck

Biological theories

Emerged with advent of Psychopharmacology in 1950's (CPZ - Thorazine: antipsychotic med) Emphasis on role of the brain vs psychodynamics as cause of MI Abnormal behavior part of disease process Source usually limbic part of the brain or NT sites of the CNS

resources for persons with SMI?

Emergency psychiatric services Group and individual psychotherapy Housing services Partial hospitalization programs (PHPs) Guardianship Community outreach programs Substance abuse treatment

Nursing application of Humanistic theories?

Emphasis on human potential and the client's strengths Prioritizing nursing actions in the nurse-client relationship

Recovery Model of Care

Emphasizes living adaptively w/chronic mental illness. Viewed as both an overarching philosophy of life for people with mental illness and as an approach to care for use by those who treat, finance, and support mental health care.

Maturational crisis situation

Erik Erickson conceptualized this process as eight stages of ego growth and development. New developmental stage is reached Leaving home, marriage, birth, death etc Old coping skills no longer helpful when a person reaches a new stage Ineffective defense mechanisms until new coping skills develop - often leads to increased tension and anxiety

Interventions for Anorexia ?

Establish trust Monitor eating patterns Be supportive rather than authoritarian or coercive

secondary nursing care for crisis intervention

Establishes intervention to prevent prolonged anxiety from diminishing personal effectiveness and personality organization

Generalized Anxiety Disorder (GAD)

Excessive anxiety or worry about numerous things Lasts for 6 months or longer Common worries are inadequacy in interpersonal relationships, jobs, finances. Sleep disturbance is common

Nontherapeutic Communication Techniques

Excessive questioning (particularly close-ended) Giving approval or disapproval Giving advice Asking "why" questions

Risk factors for Delirium?

Existing cognitive impairments polypharmacy (drug vacation - take away all meds and restart one at a time to see what is still necessary)

Bipolar II is more common in males or females?

FEMALES

Genetic and environmental factors on anger and aggression

Family dynamics affect dispositions of infants Watching violent movies, video games, abusive household

CH 29. The emergency department nurse planning care for a rape survivor must realize that the emotional reaction displayed by many rape victims during the initial assessment and treatment is which emotion? A. Fear B. Eagerness C. Aggression D. Disinterest

Fear -Rape is an act of violence, and sex is the weapon used by the perpetrator. Rape engulfs its victims in fear and anxiety, resulting in withdrawal for some and causing severe panic reactions in others. After being traumatized, the person who has been raped often carries an additional burden of shame, guilt, fear, anger, distrust, and embarrassment. None of the other options are generally associated with this form of trauma

Stages of Anger and Aggression

Feeling of vulnerability > Uneasiness> Anxiety > Anger > Aggression > Violence

CH 28. An elderly client pays the bills because she fears that her family will make her live elsewhere if she doesn't "help out." The nurse assesses as the client may be at risk for what form of abuse? A. Neglect B. Physical violence C. Psychological D. Financial abuse

Financial maltreatment - Financial maltreatment occurs when the perpetrator takes financial advantage of the elderly person, often through the use of subtle threats of what unpleasant or frightening outcome will occur if the elder does not supply funds. None of the remaining options is as directly associated with money as is the correct option

Crisis Assessment Guidelines

First assess for suicidal or homicidal ideations or gestures Then assess: - Perception of precipitating event - Situational supports - Personal coping skills - Self assessment

Flooding (Behavioral Therapy)

Flooding - exposes pt to a large amount of undesirable stimulus in an effort to extinguish the anxiety response

Severe Anxiety

Focus on 1 detail or many scattered Cannot problem solve 'sense of impending doom' More s/s

CH 28. When there is reason to suspect that a child is being abused, the nurse must initially implement which intervention? A. Call the local police to report it. B. Follow agency policy for reporting. C. Confront the parent or parents. D. Question the child to obtain proof.

Follow agency policy for reporting. - Nurses are mandated reporters of child abuse. They must follow the rules set forth by the state regarding the steps to take to report child abuse.

Prodromal phase of schizophrenia

Forewarning symptoms Can occur as early as 1 month to more than a year before a psychotic break. includes mild changes in thinking, reality testing, and mood, eccentric or odd speech and thought, anxiety, obsessive thoughts, and compulsive behavior, deterioration in concentration, school/job performance, and social functioning and distressing thoughts

Peplau Interpersonal Theory

Foundation of psychiatric nursing as specialty - major paradigm shift in nursing Shift from medical treatment to interpersonal model of relationships in nursing Nurse-patient relationship is foundation of nursing practice Identified levels of anxiety - mild, moderate, severe, panic Importance of communication

Psychoanalytic theorists?

Freud Erikson

Early Stress-Response Theories - Hans Selye (1907-1982)

General Adaptation Syndrome (GAS) - expanded theory of flight-or-flight

Early Stress-Response Theories - Hans Selye

General Adaptation Syndrome (GAS) - expanded theory of flight-or-flight Three stages 1. Alarm 2. Resistance 3. Exhaustion

CH 32. The mother of a client with severe, persistent schizophrenia tells the nurse, "My son has slipped so far away from me over the past few years. We really don't have a relationship anymore. I miss him." The nursing diagnosis that best describes the mother's feelings using which term? Grieving Powerlessness Caregiver role strain Ineffective coping

Grieving - The mother is mourning the loss of her son as she formerly knew him. Grief is a common experience for families with mentally ill members. The statement does not support any of the other options

Culture

Groups with shared beliefs, values, and practices Influences their thinking and behavior

pts with borderline PD are most commonly in ?

HOSPITALS

CH 28. The nurse performing the assessment of a wheelchair-bound client suspects that his wife's explanation of how he sustained facial contusions and a broken nose may not be entirely truthful. The nurse should implement which assessment intervention? A. Confront the wife with the suspicion that her husband's injuries are the result of abuse. B. Have the wife wait in the waiting room so her husband can be interviewed in private. C. Report the husband's injuries to the police and ask for a confidential investigation. D. Document the suspicion and follow a policy of "wait and see" whether he returns again.

Have the wife wait in the waiting room so her husband can be interviewed in private. - Suspected victims of abuse should always be interviewed in private. If the perpetrator is in the room, the victim cannot speak freely. Confronting the wife would likely cause her to be uncooperative and increase tensions. Neither of the other options address the assessment interview process.

Assessing coping styles

Health sustaining habits (e.g., medical compliance, proper diet, relaxation) Life satisfactions (e.g., work, family, hobbies, humor, art, nature) Social supports Effective and healthy responses to stress

Mild anxiety

Heightened perception Can problem solve Restless, irritable, mild tension relieving behaviors

complicated grieving

History of loss, lack of support, unpredicted death; can have prolonged depression (6+months: need referral to get help - beyond healthy grieving)

what defines major depressive disorder?

History of one or more major depressive episodes No history of manic or hypomanic episodes Symptoms interfere with social or occupational functioning

Measuring stress

Holmes and Rahe -Social readjustment rating scale -Measures level of positive and negative stressful life events over a 1-year period

HEADSSS Assessment for Adolescents

Home environment (relations w parents and siblings) Education and employment (school performance) Activites Drug, school, or tobacco use Sexuality (whether the patient is sexually active, , practices safe sex, or uses contraception) Suicide risk or symptoms of depression or other mental disorder Safety (how safe does the patient feel at home and school, wear a safety belt, or engage in dangerous or risky behaviors)

Patient rights in treatment settings

Hospitalized patients retain their rights as citizens Patient's need for safety must be balanced against patient's rights as a citizen Mental health facilities have written statements of patient's rights and applicable state laws

Bipolar II

Hypomanic episodes alternating with major depression

Cyclothymia

Hypomanic episodes alternating with minor depressive episodes (at least two years in duration

is ECT usually conducted as inpatient or outpatient?

INPATIENT

Main symptoms of depression in children?

IRRITABILITY rather than saddness

Populations at Risk of Mental Illness and Inadequate Care

Immigrants Refugees Cultural "minorities"

The nurse suspects child abuse. What important documentation is needed?

Important elements of the documentation of findings from the initial assessment include: (1) Verbatim statements of who caused the injury and when it occurred (2) A body map to indicate the size, color, shape, areas, and types of injuries, with explanations (3) Physical evidence of sexual abuse, when possible

Important considerations for nurse for pt with antisocial PD?

Important to set clear limits Communicate with staff - which precautions to take and around which patients DOCUMENT!!

Severe Alzheimer's Disease

In advanced Alzheimer's disease, most of the cortex is seriously damaged. The brain shrinks dramatically due to widespread cell death. Individuals lose their ability to communicate, to recognize family and loved ones and to care for themselves.

Sumblimation defense mechanism

In psychology, sublimation is a mature type of defense mechanism, in which socially unacceptable impulses or idealizations are transformed into socially acceptable actions or behavior, possibly resulting in a long-term conversion of the initial impulse.

Early stages of dementia involves what pathologic features?

In the earliest stages, before symptoms can be detected with current tests, plaques and tangles begin to form in brain areas involved in: - Learning and Memory - Thinking and Planning

Dissociative amnesia dissociative disorder

Inability to recall important personal information, often of a traumatic or stressful nature

Toxic effects of MAOIs

Increase in BP development of intracranial hemorrhage hyperpyrexia convulsions coma and death

Advanced Practice Interventions for abuse

Individual psychotherapy Family psychotherapy Group psychotherapy

nursing diagnoses for anger and aggression

Ineffective coping Stress overload Risk for self-directed violence Risk for other-directed violence

Neurotransmitter effect of TCAs?

Inhibit the reuptake of norepinephrine and serotonin

CH 32. The clinical nurse specialist should suggest which cognitive intervention initially for a client experiencing auditory hallucinations? Seclusion when escalation begins Physical restraints when the client is disruptive Initiating a distracting technique Giving as-needed medication for anxiety

Initiating a distracting technique - Strategies have been successfully applied to treat hallucinations, delusions, and negative symptoms, making cognitive interventions an evidence-based practice. For example, distraction techniques can be taught when auditory hallucinations occur, such as listening to music or humming. The remaining options should only be considered when less restrictive interventions, like distraction, prove to be ineffective and the client is at risk for injury to self or to others.

Outcome identification target areas of Alzheimer's Disease

Injury Communication Agitation level Caregiver role strain Impaired environmental interpretation: chronic confusion Self-care needs

Voluntary admission

Inpatient care sought by a patient or patient's guardian through a written application to the facility. have the right to request and obtain release

CH 29. Which statement would be an appropriate long-term outcome for a rape client? Integrate of the rape event and resumption of an optimal level of functioning. Identify and develop coping skills necessary to reduce level of anxiety. Appropriately blame the rapist rather than blaming herself for the situation. Repress feelings of shame, embarrassment, and self-blame.

Integrate of the rape event and resumption of an optimal level of functioning. - This is the ideal long-term result of treatment for rape trauma syndrome, that life will go on and the client will return to the usual pre-trauma level of functioning. Repressing is not a healthy coping mechanism. While not inappropriate, the remaining options are not long-term goals for such a client

Define violence

Intentional use of force to cause harm or potential harm to another person.

Immune Stress Responses

Interaction between nervous system and immune system during alarm phase of GAS Negatively affects body's ability to produce protective factors

How do you want to interview the child?

Interview parent and child separately and together

CH 29. Anticipatory teaching for a rape victim should include information addressing what common survivor problem? Denial of the event Headaches and fatigue Shock and numbness Intrusive thoughts

Intrusive thoughts - Just as in posttraumatic stress disorder, intrusive thoughts haunt the rape victim in the weeks and months during which long-term reorganization is occurring. Knowing that this is a common occurrence is reassuring to the client, who often is frightened by the symptom. While the other symptoms can occur they are not commonly associated with rape recovery

Primary Dementia

Irreversible Progressive Not secondary to any other disease - it is its own disease Example: Alzheimer's disease (60% of all dementia)

who discovered Dialectal behavioral therapy?

LENEHAN

Dialectal Behavior Therapy (DBT)

LENEHAN helps patients adapt a realistic and positive sense of self, regulate emotions, learn adaptive skills for solving problems, and correct dichotomous thinking

Cultural knowledge

Learn by attending cultural events and programs,Forge friendships with diverse cultural groups,Learn by studying, Learning cultural differences helps nurse.Establish rapport,Ask culturally relevant questions,Avoid cultural insensitivity

assessment for sexual assault

Level of anxiety Type or Style of Coping mechanisms Available support systems Signs and symptoms of: Emotional trauma Physical trauma

Neuroleptic malignant syndrome (NMS) (schizophrenia)

Life threatening medical emergency !!! S/S: - Severe muscle rigidity - hyperpyrexia - diaphoresis(main difference with anticholinergic toxicity) - tachycardia, - Stop med, transfer to med unit STAT, decrease temp, hydrate - Early detection increases chance of survival - pay attention to symptoms.

Dual-Process Model of Coping with Bereavement

Loss-oriented stressors - concentrating on the loss experience, feeling the pain of grief, remembering, and longing - stressed by the actual loss, need help coping with loss Restoration-oriented stressors - overcoming loneliness, mastering skills and roles once performed by the deceased, finding a new identity, and facing practical details of life - time has passed and immediate pain is gone, and person is now stressed about how to carry on/take over roles

GAF scoring

Low functioning - 1-30 /inpatient; Mid level functioning - 31-69 /outpatient; High functioning - 70-100 /healthy

Serotonin and anxiety and aggression

Low levels related to impulsive aggression - may incorporate antidepressant in treatment plan

Bipolar I is more common in males or females?

MALES

Maintenance blood level of Lithium?

Maintenance Blood Level: 0.4 to 1.3 mEq/L

what is the function of the brain?

Maintenance of homeostasis Regulation of autonomic nervous system (ANS) and hormones Control of biological drives and behavior Cycle of sleep and wakefulness

Is dementia reversible?

Majority of dementias are irreversible

Important dietary consideration when taking Lithium?

Make up is in the salt family ***need to maintain correct/normal SALT & WATER INTAKE ***

what does rehabilitation focus on?

Managing patients' deficits Helping them learn to live with their illness

An individual with major depression has no history of?

Manic or hypomanic episodes

CH 32. Which statement best reflects the way clients who are severely and persistently mentally ill generally perceive how others in the community see them? Many feel stigmatized and alienated. Most feel undersupported by family and friends. A large number are intensely hostile toward others. The majority are incapable of such self-reflective thought.

Many feel stigmatized and alienated. - Studies have shown that many clients experience stigmatization, alienation, loss of relationships, and loss of vocational opportunities. While some clients may have the perceptions described in the other options, none are as generally expressed as feeling stigmatized and alienated.

Physical appearance of pt with bulimia?

May not appear physically or emotionally ill Often slightly above or below ideal body weight

Medications for Intermittent Explosive Disorder?

May use SSRI's, antipsychotics (sedative nature) or mood stabilizers to reduce outbursts

Measuring stress Social Readjustment Rating Scale (Holmes and Rahe; 1967, 1978, 1997)

Measures level of positive and negative stressful life events over a 1-year period. Each life change event is assigned a weight depending on its severity.

Multidisplinary treatment team

Members of each discipline are responsible for gathering data and participating in the planning of care Treatment plan or clinical pathway provides a guideline for patient's care during hospital stay

Function of Cerebrum?

Mental activities Conscious sense of being, Emotional status, Memory, Control of skeletal muscles - movement, Language and communication

Pharmacological Interventions Treatment of Opioid Addiction

Methadone (Dolophine) - Synthetic opiate blocks craving for and effects of heroin LAAM (l-α-acetylmethadol) - An alternative to methadone Naltrexone (ReVia) - Antagonist that blocks euphoric effects of opioids Clonidine (Catapres) - Effective somatic treatment when combined with naltrexone Buprenorphine (Subutex)* - Blocks signs and symptoms of opioid withdrawal

Stage 3 Mild Cognitive Impairment

Mild cognitive decline - may be detected problems with: - word finding, memory, social or work tasks - Memory - Planning, organizing

Health teaching and health promotion for anger and aggression

Model appropriate responses and ways to cope with anger Teach pts a variety of methods to appropriately express anger Educate on coping mechanisms, de-escalation techniques, and self soothing skills

Modeling (behavioral therapy)

Modeling - therapist or significant other acts as a role model to demonstrate appropriate behavior in a feared situation.

Stage 4 Mild Cognitive Impairment

Moderate cog decline (Early AD) Problems with: - Challenging mental arithmetic - Complex tasks - Moody or withdrawn

Stage 5 : Alzheimer's Disease

Moderately Severe Decline (Moderate AD) Noticeable memory gaps Need help with day to day activities such as Choosing clothes,

Second line interventions for major depression?

Monoamine Oxidase Inhibitors (MAOIs) - Tyramine = high BP episode/danger Electroconvulsive Therapy (ECT)

Affective symptoms of schizophrenia (negative)

Mood may be: depressed, elated, erratic, hostile

Examples of Opiates?

Morphine Heroin Codeine Fentanyl Methadone Meperidine

What is significant about personality B type disorders?

Most difficult and challenging disorders to treat

Psychopharmacology: Alzheimers ~ Aricept

Most prescribed med for Alzheimer's, slows down deterioration in cognitive functions but without the potentially serious liver toxicity attributed to tacrine (other medication). Diarrhea, nausea are common side effects; nightmares (may be given during day)

CH 29. Which statement reflects a truth about rape? Some women want to be raped. Rapists are oversexed. Most rapes are planned. Most women are raped by strangers.

Most rapes are planned. - Many myths about rape exist. Most rapes are not impulsive, spur-of-the-moment acts, but are carefully planned and orchestrated. All the remaining options are common myths about the act of rape

Does dementia involve change in LOC?

NO - No Change in Consciousness - unlike with delirium where you may have changes over the course of a day

Medications for oppositional defiant disorder?

NO MEDICATIONS

Pharmacological pain Tx for aging adult?

NO OPIODS! MAY CAUSE FALLS - NSAIDS used more often

biological epidemiology of anxiety disorders

NT's that regulate anxiety: epinephrine, norepinephrine, dopamine, seratonin, and gamma-aminobutyric acid (GABA). possible abnormality of benzodiazepine receptors - may prevent effect of NT GABA which has a calming effect. Repeated stress or trauma can alter the release of NT's, and change the anatomy of the brain.

pharmacological treatment for alcohol abuse?

Naltrexone (ReVia) - Reduces or eliminates alcohol craving Acamprosate (Campral) - Helps client abstain from alcohol Topiramate (Topamax) - Works to decrease alcohol cravings Disulfiram (Antabuse) - Alcohol-disulfiram reaction causes unpleasant physical effects - Works similarly to aversion therapy

Moderate Anxiety

Narrowed perception, 'selective inattention' Can problem solve Increased physical s/s - pulse, h/a, GI, shaking

what are some effects of incest on a person?

Negative self-image Depression Eating disorders Personality disorders Self-destructive behaviors Substance abuse

CH 28. What distinction can be made between abuse and neglect? A. Neglect occurs in the psychological domain; abuse occurs in the physical domain. B. Neglect is always physical; abuse can be verbal, physical, sexual, or emotional. C. Neglect is perpetrated against children; abuse victims can be children or adults. D. Neglect is a failure to provide; abuse is a failure to control aggression.

Neglect is a failure to provide; abuse is a failure to control aggression. -Neglect is failure to provide necessary care, and abuse is physical maltreatment

treatment issues with those with SMI?

Nonadherence Anosognosia Medication side effects Treatment inadequacy Residual symptoms Relapse, chronicity, and loss

Clinical presentation of pt with Bulimia?

Normal to slightly low weight Dental caries, erosion Parotid swelling Gastric dilation and/or rupture Calluses and/or scars on hands Peripheral edema Muscle weakness Abnormal K+ and Na values- why? EKG changes Cardiomyopathy

Ethnocentric tendencies

Nurses influenced by their own professional and ethnic cultures Ethnocentrism Cultural imposition does not promote client health and well-being

obssessive-compulsive disorders (OCD)

Obsessive-compulsive disorder Body dysmorphic disorder Hoarding disorder Hair pulling and skin picking disorders

Phase I - Acute phase of schizophrenia

Onset or exacerbation of symptoms with loss of functional abilities hallucinations, delusions, apathy, social withdrawal, diminished affect, anhedonia, disorganized behavior, impaired judgement, and cognitive regression. increased support or hospitalization required

Specific behavioral therapy treatments for child and adolescents with impulse control disorders?

Operant Conditioning Point and Level System

What are the 3 main impulse control disorders?

Oppositional Defiant Disorder Intermittent Explosive Disorder Conduct Disorder

Comorbidity of Conduct disorder?

Oppositional defiant disorder might come first and may be mild form of CD ADHD, learning disabilities, substance use May be confused with bipolar disorder

Peplau 3 stages of relationship

Orientation Phase - Rapport, role & responsibilities, contract, confidentiality, terms of termination, awareness of transference, countertransference, goals Working Phase -Gather more data, facilitate change, interventions Termination Phase - Summarize goals achieved, discuss new plan for home, closure (exchange memories)

Delirium ~ orientation?

Oriented to person, disoriented to time and place - may be MORE comfortable with family and familiar faces around

Environmental factors for ANOREXIA?

Over-controlling parents (mothers usually) Many teenagers with eating disorders are perfectionists, A students, athletes, etc.

NONPharmacological pain Tx for aging adult?

PT nerve stimulation massage herbal remedies

panic disorder

Panic attacks are the key feature Sudden onset of extreme apprehension or fear, usually associated with feelings of impending doom. Normal functioning is suspended due to high levels of fear, perceptual field is limited, and misinterpretation of reality may occur. They may believe they are losing their mind or having a heart attack.

Clarifying communication techniques

Paraphrasing Restating Reflecting Exploring Projective questions - usually start with a "what if" to help people articulate, explore, and identify thoughts and feelings Presumption questions

environmental factors for impulse control disorders?

Parental rejection, harsh discipline, chaotic home life, large family, absent or alcoholic father,

examples of MAOIs to treat major depression

Parnate Nardil

Implementation in Phase I Acute for schizophrenia

Partial hospitalization Residential crisis centers Halfway houses Day treatment programs

What is refeeding syndrome?

Patient has been starved and must be careful not to overwork heart once they start to intake food again

When do you consider that treatment of PTSD is effective?

Patient recognizes treatment is effective Patient is able to use newly learned strategies to manage anxiety Pt. experiences no flashbacks Able to sleep without nightmares Assume work/family roles

outcome identification for Phase I Acute in schizophrenia

Patient safety and medical stabilization

2 initial goals in crisis intervention?

Patient safety: - Assess Anxiety reduction: - Listen, therapeutic communication - Initial directive/creative approaches - Identify social supports/coping skills

Behavioral theorists?

Pavlov Watson Skinner

Milieu therapy ~ Bulimia Nervosa ~ what to watch for?

Pay attention for over exercising / purging behaviors

what does an actual occurrence of violence require?

Perpetrator Vulnerable person Crisis situation

Erickson psychoanalytic theory

Personality develops through the lifespan 8 developmental stages (tasks, crisis)

Involuntary commitment

Petition & Certificate Court ordered admission without patients consent

Phase 1 of crisis

Phase 1 -problem>anxiety>usual coping mechanisms (problem solving-defense mechanisms) A person confronted by a conflict or problem that threatens his self-concept responds with increased feelings of anxiety. The increase in anxiety stimulates the use of problem-solving techniques and defense mechanisms in an attempt to address the problem and lower anxiety.

Phase 2 of crisis

Phase 2 -usual methods fail>increased anxiety>disorganization>trial & error methods if usual defensive response fails and the threat persists, anxiety continues to rise and produce rising levels of discomfort. Individual functioning becomes disorganized. Trial-and-error attempts at problem solving and restoring balance begin.

Phase 3 of crisis

Phase 3 - trial & error fails>severe anxiety or panic >fight or flight>compromising needs or redefining the situation to reach an acceptable solution

Phase 4 of crisis

Phase 4 - compromise fails>panic>possible personality disintegration, danger to self/others may occur

what are the different types of abuse?

Physical abuse Sexual abuse Emotional abuse Neglect Economic abuse

Data collected during mental health assessment

Physical: Review of systems Laboratory data Psychological/emotional: Mental status exam Suicide risk assessment Psychosocial assessment lifestyle: behavior re: family, living situation, friends, habits, coping style, stressors, ADL's religious/cultural aspects

Define "withdrawal"

Physiological changes occur when blood and tissue concentrations of drug decrease after heavy prolonged use of substance

indigenous Culture

Place significance on place of humans in natural world Basis of identity is the tribe Person an entity only in relation to others Disease - lack of harmony of individual with environment

General assessment in persons with schizophrenia

Positive symptoms Negative symptoms Cognitive symptoms Affective symptoms

Possible outcomes of Alzheimer's Disease

Possible Outcomes Remain safe Regular sleep pattern Self care needs met with optimal participation by patient Reduce anxiety

CH 29. Care planning for the rape victim is facilitated if the nurse understands that rape trauma syndrome is actually a variant of which psychiatric disorder? Posttraumatic stress disorder (PTSD) A maturational crisis A dissociative disorder Obsessive compulsive disorder (OCD)

Posttraumatic stress disorder (PTSD) - Most of those who have been raped are eventually able to resume their previous lives after supportive services and crisis counseling. However, many carry with them a constant emotional trauma: flashbacks, nightmares, fear, phobias, and other symptoms associated with posttraumatic stress disorder (PTSD). None of the other options are associated with this type of trauma

positive symptoms of schizophrenia

Presence of something that is not normally present - hallucinations, delusions, paranoia, disorganized or bizzare thoughts, behavior, or speech

primary nursing care for crisis intervention

Promotes mental health and reduces mental illness to decrease the incidence of crisis

tertiary nursing care for crisis intervention

Provides support for those who have experienced a severe crisis and are now recovering from a disabling mental state -Rehabilitation centers, sheltered workshops, day hospitals, outpatient clinics - Social and community facilities that provide structured environments that can help prevent problem situations

In the hospital, violence is most frequent in?

Psychiatric units Emergency departments Geriatric units Intensive care units

Advanced Practice Interventions (schizophrenia)

Psychotherapy Cognitive-behavioral therapy (CBT) Group therapy Medication Social skills training Cognitive remediation Family therapy

Sullivan interpersonal theory

Purpose of all behavior is to get needs met through interpersonal interactions and to decrease or avoid anxiety Social context of personality Importance of milieu

Rape-Trauma Syndrome: Long-Term Reorganization Phase reactions and defensive reactions:

Reactions likely to be experienced include: - Intrusive thoughts - Increased motor activity - Increased emotional lability - feel angry but suppressing their anger. Need therapy to manage feelings - Defensive reactions are: - Fears and phobias

CH 12. The most common course of schizophrenia is an initial episode followed by what course of events? A. Recurrent acute exacerbations and deterioration B. Recurrent acute exacerbations C. Continuous deterioration D. Complete recovery

Recurrent acute exacerbations and deterioration - Schizophrenia is usually a disorder marked by an initial episode followed by recurrent acute exacerbations. With each relapse of psychosis, an increase in residual dysfunction and deterioration occurs

Countertransference

Refers to the nurse's behavioral and emotional response to the client

Characteristics of pt with ANOREXIA

Refusal to maintain appropriate body weight Intense fear of gaining weight / becoming fat Sees self as fat Disturbance in body image Loss of menses (amenorrhea) < 85% ideal body weight Hunger is ignored, but no loss of appetite Unusual/ritualistic behaviors

Function of the cerebellum

Regulates skeletal muscle, Coordination and contraction, Maintains equilibrium

CH 32. The nurse working with a client diagnosed with severe and persistent mental illness will implement rehabilitation principles by concentrating on which intervention? Assessment on the client's deficits Reinforcing the client's strengths Reviewing earlier treatment plans for errors Considering the need to lower expectations periodically

Reinforcing the client's strengths - Although deficits are assessed and addressed, implementation of rehabilitation is dependent on reinforcement of identified client strengths. Neither of the remaining options is fundamental to the rehab process

Oppositional Defiant Disorder

Repeated and persistent pattern of having an angry and irritable mood in conjunction with demonstrating defiant and vindictive behavior.

CH 29. A sexual assault victim asks to be given "the morning-after pill" to prevent conception. The nurse does not believe in abortion. What action the nurse should take? Refer the woman for social services counseling. Report and document the request. Ask the supervising nurse to reassign the client. Ask the client to reevaluate her request after 24 hours.

Report and document the request. - The nurse's ethical beliefs should never interfere with client rights. The nurse should report and document the client's request.

Function of neurons

Respond to stimuli Conduct electrical impulses Release chemicals

Secondary Dementia?

Result of some other pathological process Example: AIDS-related dementia, viral encephalitis, hypothyroidism, alcohol-related dementia

Freuds Psychoanalytic theory

Revolutionary approach to MH with theory Personality structure & development, psychosexual stages of development, levels of awareness, anxiety, defense mechanisms Conscious, preconscious, unconscious ID, ego, superego Human development proceeds through 5 stages from infancy to adulthood

Alzheimer's Disease Nursing Diagnoses?

Risk for injury Impaired verbal communication Impaired environmental interpretation syndrome Impaired memory Confusion Caregiver role strain

compulsions

Ritualistic behaviors an individual feels driven to perform in an attempt to reduce anxiety

Include details on substance used such as ... ?

Route Quantity Time of last use Usual pattern of use

Evaluation of delirium?

SAFE ORIENTED UNDERLYING CAUSE BEING TREATED

Examples of ADHD medications

STIMULANTS - Ritalin, Concerta, Adderal

Subtypes of major depressive disorder

Seasonal affective disorder Melancholia Atypical depression Catatonic depression Psychotic depression Post partum depression

CH 28. When treatment for injuries sustained during an incident of abuse is sought from the primary physician, the client is receiving which form of treatment? A. Primary prevention B. Secondary prevention C. Tertiary prevention D. Stop-gap therapy

Secondary prevention - Secondary prevention involves early intervention in abusive situations to minimize their disabling or long-term effects. This scenario demonstrates actions only associated with secondary prevention.

3 hallmarks of eating disorders?

Secretiveness control manipulation

First line medications for Major depression?

Selective serotonin reuptake inhibitors (SSRIs) Newer Atypical Antidepressants Tricyclic Antidepressants (TCAs) - Can lead to more damage/overdose

Critical Incident Stress Debriefing (CISD) 7 phases?

Seven-phase group meeting: Introductory - meeting purpose explained. Fact- discuss facts of incident Thought - discuss first thoughts of incident Reaction- talk about worst things of incident Symptom Teaching - normality of symptoms discussed Reentry - review material discussed

Signs of SEVERE Lithium toxicity and range?

Severe (around 2.0-2.5 mEq/L) Confusion Incontinence of urine or feces Coma Cardiac arrhythmias Peripheral circulatory collapse Abdominal pain Proteinuria Oliguria (large output of dilute urine) Death

stage 6 : Alzheimers Disease

Severe Cognitive Decline (Severe AD) May have: changes in sleep, difficulty remembering personal history, trouble remembering names, need help with dressing & toileting, delusions or compulsive behavior, may wander and become lost

Social Anxiety Disorder

Severe anxiety or fear provoked by exposure to a social or a performance situation that will be evaluated negatively by others

CH 29. To provide discharge treatment and support, the nurse should realize that the most common outcome of acquaintance rape is which psychosocial dysfunction? Sexual distress Fear of men Depression Anxiety

Sexual distress - Women who have been raped by acquaintances frequently develop symptoms that prevent them from participating in normal sexual relations. Sexual distress is more common among women who have been sexually assaulted by intimates; fear and anxiety are more common in those assaulted by strangers. Depression occurs in both groups.

Side effects of SSRIs

Sexual dysfunction dry mouth sweating Weight loss in early therapy, weight gain with long-term Serotonin Syndrome Withdrawal Syndrome Sleepiness, faintness, lightheadedness GI Bleeding

CH 32. An issue for severely and persistently mentally ill clients living in the community is inadequate long-term medication monitoring by community mental health workers. What is a remedy for this problem? Discontinue antipsychotics that cause untoward side effects. Develop tools to predict relapse and assess the potential for violence. Shift follow-up from social workers to the ACT model. Use client empowerment techniques to increase client autonomy.

Shift follow-up from social workers to the ACT model. - Adequate monitoring of medication effects by the community-based health care provider is often difficult but more achievable when the client is being monitored by the assertive community treatment (ACT) model. None of the other options present an effective remedy for this problem

CH 32. A client diagnosed with a severe and persistent mental illness tells the case manager, "I think people are laughing at me behind my back. I get real upset and anxious when I have to be around others in the group home. It's better when I just stay by myself." The nurse should consider which nursing diagnosis to address the client's concerns? Acute confusion Social isolation Risk for activity intolerance Impaired comfort

Social isolation - Social isolation is aloneness experienced by the individual and perceived as imposed by others. None of the other options would be supported by the information provided in the question

what is "Grief overload?"

Start losing people at rapid rates, constantly facing death/dying amongst family and friends - exacerbated by a past history of depression

Tyramine MAOI poisoning side effects

Starts with headache, stiff neck....palpitations, increase/decrease in heart rate, chest pain, nausea, vomiting, pyrexia.

Best approach for pt with Narcissistic PD?

Stay neutral and do NOT give special privileges

Psychiatric Nursing in Outpatient and Community Settings

Strong problem-solving and clinical skills Cultural competence Flexibility Knowledge of community resources Autonomy Biopsychosocial assessment Case management Promoting continuation of treatment Teamwork and collaboration

Dissociative Fugue dissociative disorder

Sudden unexpected travel away from customary locale and inability to recall one's identity and information about some or all of the past (maybe even assuming a whole new identity)

Interpersonal theorists?

Sullivan Peplau

Characteristics of Antisocial PD?

Superficial charm Deceitful, manipulative, aggressive Violates rights of others - lack empathy Lies, cheats, exploits others Lack of guilt or remorse Impulsive, acts out Antisocial Behavior

CH 32. Severely mentally ill (SMI) clients often express a strong desire to be employed. According to the evidence-based research, what is the most effective model of employment for these clients? Vocational rehabilitation Productive employment A placement program of rehabilitation Supported employment

Supported employment - In the past, vocational rehabilitation programs required extensive evaluation procedures and training before attempting job placement. However, these programs were unsuccessful at helping severely mentally ill clients to maintain jobs. Research efforts have identified a more productive model called supported employment.

Phase II - Stabilization of Schizophrenia

Symptoms diminishing Movement towards previous level of functioning care in outpatient mental health center or partial hospitalization

Systematic desensitization (behavioral therapy)

Systematic desensitization - pt. gradually introduced to a feared object or experience thru a series of steps, from least frightening to most (graduated exposure). Pt is taught to use a relaxing technique at each step when anxiety becomes overwhelming

"Start low go slow" is a term used with what medication?

TCAs for major depression

TOXIC blood level for lithium?

TOXIC BLOOD LEVEL: 1.5 to 2.0 mEq/L

Signs and Symptoms of Delirium Tremens?

Tachycardia, diaphoresis, elevated blood pressure Disorientation and clouding of consciousness Visual or tactile hallucinations Hyperexcitability to lethargy Paranoid delusions, agitation Fever (100° F to 103° F)

CH 12. Which side effect of antipsychotic medication is generally nonreversible? A. Anticholinergic effects B. Pseudoparkinsonism C. Dystonic reaction D. Tardive dyskinesia

Tardive dyskinesia - Tardive dyskinesia is not always reversible with discontinuation of the medication and has no proven cure. The other side effects often appear early in therapy and can be minimized with treatment.

Tardive dyskinesia (1st gen. side effect)

Tardive dyskinesia - Persistent EPS side effect involving involuntary rhythmic movements after prolonged treatment, and persists after med is stopped

Other outpatient venues for psychiatric care

Telephone crisis counseling Telephone outreach Internet Telepsychiatry

assessment: interview guidelines verbal approach? (abuse)

Tell me about what happened to you. Who takes care of you? (for children and dependent elders) What happens if you do something wrong? How do you and your partner resolve disagreements? What do you do for fun? Who helps you with your children/parent? What time do you have for yourself? Ask the caregiver "what do you do for fun?"

Prevalence

The number or proportion of cases of a particular disease or condition present in a population at a given time.

Bioethics

The study of ethics related to issues that arise in health care.

CH 28. When should the nurse expect the abuse of a victim to worsen? A. The perpetrator feels he is in complete control. B. The perpetrator is feeling remorseful for being abusive. C. The victim moves toward independence from the abuser. D. The victim submits to the domination of the perpetrator.

The victim moves toward independence from the abuser. - When the abuser thinks they are losing control over the victim, the violence escalates. None of the other options are associated with the escalation of abuse by the perpetrator against the victim

Therapeutic blood level for Lithium

Therapeutic Blood Level : 0.8 to 1.4 mEq/L

Community Mental Health Centers Act

This act was passed in 1963 to provide federal funding for community mental health centers, training programs, and outpatient treatment programs

thought stopping (behavioral therapy)

Thought stopping - negative thought or obsession is interrupted.

Cluster B personality disorders experience difficulty in what 3 areas?

Thoughts and emotions Participation in interpersonal relationships Managing impulses

obsessions

Thoughts, impulses, or images that persist and recur, so that they cannot be dismissed from the mind

Abnormal Involuntary Movement Scale (AIMS) (1st gen. side effects)

Tracks TD symptoms Administration is key nursing role Try to screen people every 3 mo

trauma informed care with anger and aggression

Treat patients as if they had been traumatized in the past Disruptive patients often have histories that include violence and victimization. Focus on patients' past experiences of violence or trauma and the role of aggression currently plays in their lives

Common secondary issues to Delirium

UTI kidney/circulation problems tumors alcohol Lack of sleep sensory deprivation/overload

Adventitious Crisis Situation

Unplanned and accidental - Natural disaster - Fire, flood, tornado, earthquake... National disaster - War, terrorism, riots, plane cresh... Crime of violence - Assault, abuse, murder, rape...

Therapeutic use of self

Use personality consciously and in full awareness Attempt to establish relatedness Structure nursing interventions

Tools for enhancing communication

Using silence Active listening Clarifying questions Questions Listening with empathy

Symptoms of childhood conduct disorder

Usually in males Physically aggressive Poor peer relationships Lacks feelings of guilt or remorse Low self-esteem and tolerance for frustration

Conditional release

Usually requires outpatient treatment for a specified period to determine if the pt follows the medication regimen, can meet basic needs, and is able to reintegrate into the community.

Western Tradition

Values Identity found in individuality Mind and body separate entities Disease has a cause, and treatment is aimed at cause Time is linear Success obtained in preparing for the future

Stage 7 : Alzheimer's Disease

Very Severe Decline (Late-stage AD) May lose ability to: respond to environment, converse, control movement, smile, swallow, sit without support

Conduct disorder Behavior NOT within age-appropriate societal norms?

Violates the rights of others Aggression toward people and animals Destruction of property Deceitfulness or theft Serious violation of rules Violates rights of others

CH 28. Which statement reflects a fact about family violence? A. Ninety-five percent of abuse victims are women. B. The victim's behavior is often the cause of the violence. C. Violence occurs in families of all backgrounds. D. Alcohol and stress are the major causes of abuse.

Violence occurs in families of all backgrounds. - Family violence is a serious community health problem common among all backgrounds. None of the other statements provide accurate information about family violence.

Assessment tools for the aging adult

Visual Analogue Scales - line from worst————best and patient marks with a slash on line Wong-Baker FACES Pain Rating Scale - patient points to face that matches them Present pain intensity (PPI) rating from the McGill Pain Questionnaire (MPQ) - pick description that best describes their pain Pain Assessment in Advanced Dementia (PAINAD) scale - based on observation, not on patient's opinion/explanation (facial expression, body language, consolability)

Is inpatient admission voluntary or involuntary?

Voluntary OR involuntary

CH 10. which theorist investigated the sympathetic nervous system in animals in response to stressors. He discovered the: Fight (aggression) -or-flight (withdrawal) response A. Walter Cannon (1871-1945 B. Sigmund Freud C. Hans Selye (1907-1982) D. Harry Stack Sullivan

Walter Cannon (1871-1945)

Effect of cocaine and crack

When smoked, takes effect in 4 to 6 seconds; a 5 to 7 minute high follows, then a deep depression

intentional tort

Willful or intentional acts that violate another person's rights or property - Battery - Assault - False imprisonment - Invasion of privacy (photos, audio) - Defamation of character (slander or libel)

MENNINGER psychosocial factors

Wish to kill Wish to be killed Wish to die

If you have pt experiencing delirium ~ let them have visitors or no?

YES

can major depression include psychotic symptoms?

YES - from a depressed angle, NOT manic.

Profile of Sexual Perpetrators

Young 30% under 21 23% under 30 Under the influence of alcohol or drugs at time of offense 61% acquainted with victim

selective mutism

a condition where children do not speak due to fears of negative responses or evaluation

Hypomania

a mental state just below mania

dissociative identity disorder (DID)

a rare dissociative disorder in which a person exhibits two or more distinct and alternating personalities. Each alternate personality (alter) has own pattern of - Perceiving - Relating to and - Thinking about the self and environment

Nursing Code of Ethics

a set of principles that reflect the primary goals, values, and obligations of the profession

operant conditioning

a type of learning in which behavior is strengthened if followed by a reinforcer or diminished if followed by a punisher

5. Which observations, if documented in the medical record, would indicate that the treatment plan of a severely depressed client has been effective? a. "Slept 6 hours uninterrupted, sang with group in activities, anticipates seeing grandchild." b. "Slept 10 hours, attended craft group, stated his project was a mess, just like me." c. "Slept 5 hours, personal hygiene adequate with assistance, weight loss of 1 pound." d. "Slept 7 hours, states he feels tired most of the time, preoccupied with perceived inadequacies."

a. "Slept 6 hours uninterrupted, sang with group in activities, anticipates seeing grandchild."

Which assessment finding represents a negative symptom of schizophrenia? a. Apathy b. Delusion c. Motor tic d. Hallucination

a. Apathy

24. An appropriate short-term goal for an antisocial client with a nursing diagnosis of Ineffective coping related to use of manipulation would be a. Client will acknowledge manipulative behavior when it is called to his or her attention. b. Client will use manipulation only to get his or her needs met. c. Client will accept staff explanation that a request will be met within an hour rather than immediately. d. Staff will state fewer negative counter-transferences about client.

a. Client will acknowledge manipulative behavior when it is called to his or her attention.

4. What initial nursing intervention is appropriate to take in the immediate post-ECT treatment period? a. Monitor vital signs closely b. Repeatedly stimulate the client to respond. c. Assist the client to sit up, then ambulate. d. Begin forcing fluids

a. Monitor vital signs closely

12. What environmental conditions should the nurse ensure for G while she is experiencing sensory perceptual alterations? a. a quiet, well-lit room without glare while client experiences sensory perceptual alterations b. allowing client to sit by nurse's desk while out of bed; providing frequent rest periods in room with television or radio on c. a brightly lit room around the clock; awakenings hourly to check mental status d. a softly lit room around the clock; television on during day and evening

a. a quiet, well-lit room without glare while client experiences sensory perceptual alterations

15. A nursing intervention designed to help the client with progressive memory deficit function in his or her environment is to a. assist client to perform simple tasks by giving step-by-step directions b. avoid frustrating client by performing routines associated with activities of daily living for the client c. stimulate the client's intellectual functioning by bringing new topics, objects, etc. to the client's attention d. promote use of client's sense of humor by telling jokes or riddles and discussing cartoons

a. assist client to perform simple tasks by giving step-by-step directions

3. A depressed client who is scheduled to receive ECT this morning asks the nurse, "How is this treatment supposed to help me?" The best reply would be, "ECT a. probably increases the availability of brain neurotransmitters." b. makes you confused and you forget why you're feeling depressed." c. serves as a punishment, so your own conscience can stop punishing you." d. works by opening your mind to learning new coping skills

a. probably increases the availability of brain neurotransmitters."

6. Which is true of the SSRI's? a. they have a favorable side effect profile and efficacy. b. they are usually a last choice option c. they usually take 4-6 weeks to see full effect d. all of the above

a. they have a favorable side effect profile and efficacy.

negative symptoms of schizophrenia

absence of something that should be present. - inability to enjoy activities, social discomfort, lack of goal-directed behavior

Cyclothymia is commonly seen in what age group?

adolescents or young adults

Childhood onset of conduct disorder occurs before what age?

age 10

identification defense mechanism

an attempt to increase self-worth by acquiring certain attributes and characteristics of an individual one admires

identification: defense mechanism

an attempt to increase self-worth by acquiring certain attributes and characteristics of an individual one admires

Using Erickson theory in nursing

assessment re: developmental stages; tailor interventions to person's developmental level

How long does depressed mood last for in major depressive disorder?

at least 2 weeks

Echolalia

automatic and immediate repetition of what others say

An adult with paranoia becomes agitated and threatens to assault a staff person. Select the best initial nursing intervention. a. Tell the patient, "If you do not calm down, seclusion will be needed." b. Address the patient with simple directions and a calming voice. c. Help the patient focus by rubbing the patient's shoulders. d. Offer the patient a dose of antipsychotic medication. e. Reorient the patient to the time and place.

b. Address the patient with simple directions and a calming voice.

Which patient behavior is a criterion for mechanical restraint? a. Screaming profanities b. Assaulting a staff person c. Refusing a medication dose d. Throwing a pillow at another patient

b. Assaulting a staff person Indications for the use of mechanical restraint include protecting the patient from self-harm and preventing the patient from assaulting others.

A violent patient is restrained. What is the nurse's first priority? a. Debrief the patient b. Ensure the patient's safety c. Administer a sedating medication d. Obtain an order from the health care provider within 1 hr. if doctor is not available, call the medical doctor on call and have them sign off.

b. Ensure the patient's safety Once in restraint, a patient must be directly observed and formally assessed at frequent, regular intervals for level of awareness, level of activity, safety within the restraints, hydration, toileting needs, nutrition, and comfort.

10. Hypertensive Crisis is a serious side effect of which medication? a. Lithium b. Parnate (MAOI) c. Prozac (SSRI) d. Ativan (Benzodiazepine)

b. Parnate (MAOI)

A nurse assesses a patient diagnosed with dissociative identity disorder. Which finding would likely be part of the patient's history? a. Travel to a foreign country b. Physical or sexual abuse c. Thyroid dysfunction d. Eating disorder

b. Physical or sexual abuse

22. A client with which of the following personality disorders lacks empathy, has a sense of entitlement, violates the rights of others and impulsively acts out a. histrionic personality disorder b. antisocial personality disorder c. borderline personality disorder d. narcissistic personality disorder

b. antisocial personality disorder

9. Which of the following are examples of food to avoid when taking the antidepressant Parnate? a. oranges, grapefruits, and lemons b. avocados, figs, smoked meats c. chicken, salmon and veal d. any food with salt

b. avocados, figs, smoked meats

25. M, a client with borderline personality disorder, tells Nurse J, "You used to care about me. I thought you were wonderful. Now I can see I was mistaken. You're hateful." This outburst can be assessed as use of a. splitting b. denial c. reaction formation d. separation-individuation strategies

b. denial

19. People who are serious about suicide usually don't give clues a. true b. false

b. false

13. Which of the following would the nurse assess as an example of cognitive impairment? a. crying when the occasion calls for laughter b. inability to name a familiar object c. incontinence d. agitation

b. inability to name a familiar object

20.Which is included in the lethality assessment of suicide? a. specificity of the plan b. lethality of the plan c. availability of means d. all of the above

b. lethality of the plan

23. M, a client with borderline personality disorder, has come back from a pass having lacerated her wrists. The physician orders daily dressing changes for the lacerations. The nurse performing this care should a. encourage M to express anger b. provide care in a matter-of-fact manner c. be sympathetic and concerned d. offer to listen to M's feelings about cutting

b. provide care in a matter-of-fact manner

2. Information given to a depressed client when the client is begun on tricyclic antidepressant therapy such as Norpramin should include a. the need to have weekly blood draws to closely monitor risk for toxicity b. the fact that mood improvement may take 7 to 28 days c. instructions to restrict sodium intake to 1 g daily d. the need to maintain a tyramine-free diet

b. the fact that mood improvement may take 7 to 28 days

16. V has Alzheimer's disease. During morning care, the nursing assistant asks her, "How was your night?" V replies, "It was lovely. My husband and I went out to dinner and to a movie." The nurse who overhears this should make the assessment that V is a. demonstrating a sense of humor b. using confabulation c. perseverating d. delirious

b. using confabulation

modeling, systemic desensitization, flooding, response prevention, and thought stopping are used in what kind of therapy? A. cognitive B. cognitive-behavioral C. behavioral

behavioral therapy

cognitive theories and anxiety

believe anxiety disorders are caused by distortions in an individuals thoughts and perceptions.

Harry Stack Sullivan

believed anxiety is linked to emotional distress caused when early needs go unmet or disapproval is experienced (interpersonal theory). Also suggested anxiety is contagious, transferring from mother to infant.

SSRIs

block reuptake of serotonin

21. Which of the following is an intervention to minimize suicidal opportunity on the inpatient locked adult unit? a. A telephone crisis line b. A support group c. Suicidal precautions every 15 minutes d. Private time alone in room

c. Suicidal precautions every 15 minutes

11. G is a client with fluctuating levels of consciousness, disturbed orientation, and perceptual alterations. An important facet of nursing care for G will be a. application of wrist and ankle restraints b. avoidance of physical contact c. careful observation and supervision d. providing a high level of sensory stimulation

c. careful observation and supervision

7. Serotonin syndrome results in: a. stiffness in the patient's gait b. hypothyroidism and cardiotoxicity c. confusion and myoclonus d. catatonic muteness

c. confusion and myoclonus

1. A student nurse caring for a depressed client reads in the client's medical record: "This client clearly shows the vegetative signs of depression." What can the student expect to observe? a. suicidal ideation b. feelings of hopelessness, helplessness, and worthlessness c. constipation, anorexia, sleep disturbance d. anxiety and psychomotor agitation

c. constipation, anorexia, sleep disturbance

8. A serious side effect from the tricyclic antidepressants that warrants medication attention is: a. stomach upset b. restlessness c. severe consitpation d. dry mouth

c. severe consitpation

Child abuse perpetrators are most often a. extended family members. b. child care workers. c. the child's parents. d. strangers.

c. the child's parents. It is estimated that 80% of perpetrators are the victim's parents.

AARON BECK psychosocial factors

central emotional factor is hopelessness

Physiological stressors

cold, heat, trauma, hemorrhage, and infection

recent theories regarding suicide?

combination of suicidal fantasies and significant loss

Which conduct disorder is most severe?

conduct disorder

Using Freuds theory in nursing

consider root causes of suffering, attentive listening, value of talk sessions, transference, countertransference

Incidence

conveys info about the risk of contracting a disease by referring to the number of new cases of mental disorders in a healthy population within a given period of time.

18. F's business has gone bankrupt. His wife has filed for divorce. F has been despondent for 2 weeks. Which statement could be assessed as a covert clue to suicide? a. "Life isn't worth living." b. "I wish I were dead." c. "My family will be better off without me." d. "I have a plan that will fix everything."

d. "I have a plan that will fix everything."

14. What is considered expected behavior for a client in Stage 2 of Alzheimer's Disease? a. Short term memory impairment b. Decline in AdL's c. Increasingly labile mood and anger d. All of the above

d. All of the above

17. The initial task of the nurse who is manning the suicide telephone line is to a. assess lethality of the suicide plan b. establish rapport with the caller c. encourage alternative expression of anger d. determine whether the caller is making a crank call

d. determine whether the caller is making a crank call

Loose associations in a person with schizophrenia indicate? a. paranoia. b. mood instability. c. depersonalization. d. poorly organized thinking.

d. poorly organized thinking

low serotonin levels are related to?

depressed mood

neurobiological risk factors for schizophrenia

dopamine, serotonin, and glutamate.

Anticholinergic effects

dry mouth, blurred vision, constipation, urinary retention, tachycardia

Intermittent explosive disorder

episodes during which a person acts on aggressive impulses that result in serious assaults or destruction of property

Common behaviors of bipolar pts?

excessive - make up/hair/clothes/talking/spending grandiosity dangerous behavior/activities reduced sleep easily distracted obsessed with/overfocused on goals

agoraphobia

fear or avoidance of situations, such as crowds or wide open places, where one has felt loss of control and panic

Mental Health Parity Act

forbids health plans from placing lifetime or annual limits on mental health coverage that are less generous than those placed on medical or surgical benefits

how can you counter the adverse effects of TCAs?

give the dose at BEDTIME

immediate side effects after ECT?

headache short term memory loss confusion nausea

Transference

in psychoanalysis, the patient's transfer to the analyst of emotions linked with other relationships (such as love or hatred for a parent)

psychological stressor

include events such as divorce, loss of a job, death of a loved one. Can also include changes we consider as positive.

Apathy (negative symptom)

lack of interest in activities or beliefs that would otherwise be interesting or important

Affordable Care Act

law passed in 2010 to expand access to insurance, address cost reduction and affordability, improve the quality of healthcare, and introduce the Patient's Bill of Rights

Neuropeptides

long - term changes in cells

Psychological stressors

loss of a job divorce death of loved one

what are some examples of a serious mental illness?

major depressive disorder, bipolar disorder, schizophrenia, schizoaffective disorder, panic disorder, PTSD, borderline and and antisocial personality disorder, and OCD

least restrictive alternative doctrine

mandates that the least drastic means be taken to achieve a specific purpose

primary prevention for abuse

measures taken to prevent the occurrence of abuse

Pts with Dementia experience difficulties in ... ?

memory thinking comprehension

Stage 2 Mild Cognitive Impairment

mild cognitive decline Aware of memory loss: no s/s detected in medical exam

define "panic"

most extreme level of anxiety and results in markedly dysregulated behavior. May lose touch with reality.

Sexual assault response team (SART)

multidisciplinary team approach to caring for victims of sexual assault. members include nurses, physicians, attorneys, social service workers, advocates, mental health professionals, forensic lab personnel

neologism

new word or expression

Stage 1 Mild Cognitive Impairment

no impairment

Dissociative disorders

occur after significant adverse experiences/traumas, and individuals response to stress with severe interruption of consciousness. Unconscious defense mechanism.

Define "Mild Anxiety"

occurs at the normal experience of every day living and allows an individual to perceive reality in sharp focus. A person will see, hear, and gasp more info, and problem solving becomes more effective.

Schizophrenia sub types

paranoid disorganized catatonic residual undifferentiated

Conduct disorder

patterns of behavior in which the rights of others or basic social rules are violated

Perception and stress

perception of the stressor that determines the person's emotional and psychological reactions to it.

post-traumatic stress disorder (PTSD)

persistent reexperiencing of a highly traumatic event that involves actual or threatened death or serious injury to self or others.

define "severe anxiety"

person may focus on one particular detail or many scattered details and have difficulty noticing what is going on in the environment, even when another person points it out. Learning and problem solving are not possible.

define "moderate anxiety"

person sees, hears, and grasps less info and may demonstrate selective inattention.

individual temperament and stress

personal outlook on life that combine to form a unique personality with specific strengths and vulnerabilities

Adjustment disorder

precipitated by a stressful event that is not considered traumatic. Responses to event include depression, anxiety, and conduct disturbances. Symptoms include guilt, depression, anxiety, anger, social withdrawal, work/academic inhibition.

Who conducts ECT?

psychiatrist anesthesiologist nurse

Using Sullivans theory in nursing

psychotherapeutic environment as treatment tool, mutuality, empathy

define "fear"

reaction to specific danger.

Alogia (negative symptom)

reduced fluency and productivity of thought and speech

Perseveration

repetition of phrase or behaviors (tell you the same story 10 times)

clang associations

rhythmic patterns associated with psychotic speech

Nursing application of behavioral theories?

risk of this model - At times discredits all other theories and influences of MI (social, environmentat, cultural etc). Nurses administer & monitor med effects, S/E

Barriers to Accurate Pain Assessment with aging adults

sensory/perception impairment unable to communicate pain belief that pain should be kept private

Informal admission

sought by patient Least restrictive of all admissions Patient is free to leave or stay

cognitive symptoms of schizophrenia

subtle or obvious impairment in memory, attention, thinking. judgement, or problem solving

Behavior theories and anxiety

suggest anxiety is a learned response to specific environmental stimuli (classical conditioning) - example - a boy who is anxious in the presence of his abusive mother. He then generalizes this anxiety as a response to all women.

psychodynamic theories on anxiety disorders

suggest that unconscious childhood conflicts are the basis for future symptom development

what can happen when schizophrenic pt. stops taking meds?

symptoms can come back much worse

Affective symptoms of schizophrenia

symptoms involving emotions and their expression

How long does Lithium Carbonate take to take effect?

takes 2-3 weeks; will use antipsychotics in the meantime to stabilize

BIGGEST contraindication with MAOIs?

taking MAOI with TYRAMINE.

what increases the risk of anticholingeric toxicity?

taking multiple medications with ACh side effects

Somatization (defense mechanism)

tendency to experience and communicate psychological distress in the form of somatic symptoms and to seek medical help for them. More commonly expressed, it is the generation of physical symptoms of a psychiatric condition such as anxiety.

Unconditional release

termination of a patient-institution relationship per medical doctor or court orders

Phase III - Maintenance of schizophrenia

the condition has stabilized and a new baseline is established. positive symptoms are absent or diminished, but negative and cognitive symptoms continue to be a concern.

Grief

the reaction to loss Includes depressed mood, insomnia, anxiety, poor appetite, loss of interest, guilt, dreams about the deceased, poor concentration

In 20-30% of pts, ADHD medication can cause ... ?

the stimulant is ineffective or causes tics, mood swings, insomnia and OCD symptoms

How to treat Delirium?

treat underlying condition

Confabulation

unconscious attempt to maintain self-esteem

Anticholingeric side effects

urinary retention, dilated pupils, constipation, tachycardia, dry mucous membranes, cognitive impairment

Humanistic Theories

view personality with a focus on the potential for healthy personal growth

profile of those at highest risk of suicide?

white males older than 75 years of age at highest risk

How close does nurse have to stay for 1:1?

within arms reach

Choreoathetoid movements (1st gen. side effect)

writhing, wormlike movements - especially of tongue and face Lip-smacking In & out tongue protrusion IRREVERSIBLE! PREVENTION IS KEY MEASURE!

can brain structure abnormalities play a role in risk for schizophrenia?

yes

is schizophrenia genetic?

yes. 80% of risk comes from genetic and epigenetic factors.

is normal anxiety okay?

yes. it is necessary for survival

Decade of the Brain

•1990 - 2000 by Pres. George H. W. Bush •Years dedicated to new studies & to make legislation/general public aware of advances that were made in neuroscience/brain research


Ensembles d'études connexes

3th mid practice quiz. ch #44, and 45

View Set

MUSIC CH 49 Mythical Impressions DEBUSSY

View Set

Chapter 27: Safety, Security, and Emergency Preparedness

View Set

Chapter 4 - Health Record Content & Documentation

View Set

ECON131 Chapter 5: Introduction to Macroeconomics

View Set

Nutrition and Drugs Final Ch. 10

View Set

Module #21-Chapter 21- Bureaucracy

View Set

Chapter 5 - Religion, Rituals, and Health

View Set